التخطي إلى المحتوى الرئيسي

الرد المبين على افتراءات مقالة أخطاء القرآن العلمية والردود الصلعمية الفاشلة عليها




هنا تجدون ردود على مجموعة أكاذيب وافتراءات ضخمة جمعها اعداء الاسلام، في سبيل الافتراء على القرآن الكريم والسنة النبوية الشريفة والادعاء انهما يحتويان على أخطاء علمية.

اسم مجموعة الافتراءات والأكاذيب

" أخطاء القرآن العلمية والردود الصلعمية الفاشلة عليها "

وقد أبقيت على كل افتراء واتبعته بردٍ يليه . راجيًا أن يكون ذلك في ميزان حسناتي ، ولا تنسوني من دعائكم

(محمد سليم مصاروه - صيدلي وماجيستير في علوم الأدوية )





أخطاء القرآن العلميّة و الردود الصلعميّة الفاشلة عليها :

الافتراء:

1 - زوجيّة الأشياء في القرآن :

مِنْ كُلِّ شَيْءٍ خَلَقْنَا زَوْجَيْنِ لَعَلَّكُمْ تَذَكَّرُونَ / الذاريات : 49

وَمِنْ كُلِّ الثَّمَرَاتِ جَعَلَ فِيهَا زَوْجَيْنِ اثْنَيْنِ / الرعد : 3

حَتَّى إِذَا جَاءَ أَمْرُنَا وَفَارَ التَّنُّورُ قُلْنَا احْمِلْ فِيهَا مِنْ كُلٍّ زَوْجَيْنِ اثْنَيْنِ / هود : 11

و اذا طبقنا هذه الآبات وجدنا فيها شيئاً من التناقض مع الوقائع المكتشفة علميّاً

فمثلاً النحل ليس زوجان فقط كنوع فهناك الملكة و الذكور و الشغّالات..و كذلك هي اغلب الكائنات ذات أنواع كثيرة جدّاً من بري و استوائي و قطبي .... الخ هذا عدا المخنّثين فيهم فلا يمكن حصر تلك الكائنات بزوجين فقط لكل شيء منها

- المخلوقات أحادية الجنس

هناك الكثير من الكائنات وحيدة الجنس أو الخليّة تتكاثر بنفسها دون أن يوجد لها نوع مذكّر و نوع مؤنّث !!

و الأمثلّة كثيرة جدّاً منها

( - خيار البحر Cnemidophorus - Branta Leucopsis - varanus komodoensis - sea cucumber

- Marmorkrebs قرش المطرقة bonnethead shark - وأغلب الرخويّات ) ...

بالاضافة الى الكائنات عديمة الجنس كالطحالب و جميع أنواع الدرنيات

و لا يمكن لأي احد نقض هذه الحقيقة المثبتة مهما حاول تضييع النص كما يفعل من يسمون انفسهم رادي شبهات !!



الرد :



الادعاء اعلاه مبني على تلاعب في معنى كلمة " زوجين " فصاحب الشبهة يحصر معنى كلمة زوجين فقط بوجود ذكر وأنثى بينما اننا لو فحصنا في المعجم معنى كلمة " زوجين " لوجدنا أن معناها نوعان ، ضربان ، صنفان .. والنوعية يمكن ان يكونا ذكر وأنثى او الشيء ونقيضه مثل ( السالب والموجب ) او الشى ومقابله ( مثل النهار والليل ) . والقرآن الكريم يقول ان كل مخلوق فيه صفة الزوجية

وَمِنْ كُلِّ الثَّمَرَاتِ جَعَلَ فِيهَا زَوْجَيْنِ اثْنَيْنِ / الرعد : 3

وهذا صحيح لأن تكوين الثمار يتوقف على حدوث عمليتي التلقيح والاخصاب والتي هي عبارة عن اتحاد حبوب اللقاح بالبويضات الموجودة في مبيض الزهرة . والثمار تحتوي على البذور والبذور نفسها حين تنفصل عن الثمار ستنمو الى شجره مزهره تنقسم الى عدة انواع هناك شجر ينتج أزهار تحتوي على مركب أنثوي وذكري في نفس الزهرة

وهناك أشجار فيها نوعين شجر ينتج أزهار انثويه وشجر ينتج أزهار ذكريه



وهناك نوع ثالث من الشجر ينتج أزهار ذكريه منفصله وبنفس الوقت أزهار انثويه على نفس الشجرة

وكما نرى فالآية الكريمة (وَمِنْ كُلِّ الثَّمَرَاتِ جَعَلَ فِيهَا زَوْجَيْنِ اثْنَيْنِ /الرعد : 3) تتطابق مع علوم النبات وتقول بان كل ثمره ناتجه عن تلقيح حادث من نوعين من الزهور ، والحقيقة ان هذه الآية عباره عن اعجاز علمي ذكره القرآن مئات السنين قبل تطور علوم النبات ! وبالنسبة للادعاء حول النحل ففيه الذكر والأنثى سواءً كان ذلك ملكه ، عاملات او ذكور وليس هناك اي تناقض مزعوم!

بالنسبة للكائنات وحيده الجنس والتي تتكاثر بنفسها فهي تحتوي على كلا نوعي خلايا الجنس ، حيوانات منويه وبويضات ، وتفرزها ويحدث تلقيح بين كلا نوعي الخلايا وهكذا ينتج النسل وخيار البحر sea cucumber مثال على ذلك فهو يتكاثر بهذه الطريقة

وهذا ينطبق على الآية الكريمة التي تقول ان الله خلق من كل شيء زوجين

(ومِنْ كُلِّ شَيْءٍ خَلَقْنَا زَوْجَيْنِ لَعَلَّكُمْ تَذَكَّرُونَ / الذاريات : 49) ،



لاحظوا ان القرآن الكريم لم يقل ان كل الكائنات لها ذكر وأنثى مثلما يَدَّعي صاحب الافتراء ! والزوجين في حالة ( خيار البحر والكائنات التي تتكاثر ذاتياً ) موجودان في نفس الكائن الحي ، كذلك بالنسبة للكائنات احادية الخلية والتي ليس فيها مذكر ومؤنث مثل البكتيريا والڤيروسات فهي تضاعف حمضها النووي وتنقسم كل مره الى خليتين اي زوجين بالضبط مثلما قالت الآية الكريمة (ومِنْ كُلِّ شَيْءٍ خَلَقْنَا زَوْجَيْنِ لَعَلَّكُمْ تَذَكَّرُونَ )







الافتراء:

2 - الفصل بين الشمس و النجوم في القرآن !!

" يَسْجُدُ لَهُ مَنْ فِي السَّمَاوَاتِ وَمَنْ فِي الْأَرْضِ وَالشَّمْسُ وَالْقَمَرُ وَالنُّجُومُ .." الحج :18

والشمس والقمر والنجوم مسخرات بأمره ( الأعراف: 54 )

وغيرها الكثير من الآيات التي فصلت بين الشمس و النجوم والتي هي كغيرها من النجوم والتفريق بينهما ما هو الّا أحد التصوّرات القديمة للكون في حين نرى تركيز القرآن على الأرض وما حولها متجاهلين الكون الواسع الهائل المليء بمليارات النجوم و الكواكب و المجرّات التي لم يأت بأي ذكر لها حتى يثبت بأنه من خالق يرى الكون وكتاب معجز لا من صنع بشر بنظرة بشريّة قديمة له !!



الرد

ادعاء مضحك وسخيف !

القرآن الكريم يخاطب البشر على الكرة الارضية ولذلك يذكر الارض وهو المكان المهم لمعيشتهم، وبنفس الوقت الشمس حيوية لحياة البشر وكل نهار يرونها، فلذلك ذكرها القرآن الكريم لكن ذلك لا يعني ان الشمس ليست نجماً !

ولم يقل القرآن بأن الشمس ليست نجمًا بل هي نجم مهم وحيوي ولذلك ذكرها بالاسم.

والقران ذَكَر الكون كله حين ذكر السماء الاولى والسموات السبع وآيات القران الكريم ما فتئت تدعو البشرية الى التأمل في الكون والبحث فيه .

ومن يدعي ان القران الكريم من صنع البشر فليتفضل وليؤلف لنا شيئاً مثله كي  يقنعنا !

الافتراء:

3 - الأخطاء العلميّة بعمل الخلق بالقرآن

وَأَوْحَى رَبُّكَ إِلَى النَّحْلِ أَنِ اتَّخِذِي مِنَ الْجِبَالِ بُيُوتًا وَمِنَ الشَّجَرِ وَمِمَّا يَعْرِشُونَ

ثُمَّ كُلِي مِنْ كُلِّ الثَّمَرَاتِ فَاسْلُكِي سُبُلَ رَبِّكِ ذُلُلًا يَخْرُجُ مِنْ بُطُونِهَا شَرَابٌ مُخْتَلِفٌ أَلْوَانُهُ فِيهِ شِفَاءٌ لِلنَّاسِ إِنَّ فِي ذَلِكَ لَآَيَةً لِقَوْمٍ يَتَفَكَّرُونَ النحل / 68 - 69

بغض النظر ان اعتبرنا أن الأزهار ثمار !! فهل العسل يخرج من بطن النحل !!؟؟ الجواب : لا

فالعاملات من النحل تجمع " النكتار " ( 20% منه عسل ) داخل بطن النحلة ضمن معدة خاصة تسمى معدة العسل و يمر عبر معي عاملات النحل ليخرج من فمها بعد اختلاطه بعدد من الأنزيمات و البروتينات وبعد أن تضع النحلة نكتار العسل داخل الخلايا الشمعية التي تصنعها تقوم بتجفيفه بفمها حتى يتحوّل النكتار الى عسل خالص !!

http://animals.nationalgeographic.com/animals/bugs/honeybee/

http://en.wikipedia.org/wiki/Honey#Formation

http://www.fordshoneyfarm.com/honeymade.html

وتستغرق عمليات التحول من الرحيقِ الزهري الذي تجمعه عاملات النحل فقط إلى العسل حوال يوم كامل

العسل اول من اكتشف فائدته و بدأ باستخراج الأدوية منه هم المصريين القدماء - الفراعنة -و الصينيين القدماء ومن قبل الإسلام بأكثر من الف وستمائة عام ليس نبي المسلمين كما يدعي ادعياء الاعجاز العلمي !!

http://pharmacy.ksu.edu.sa/ar/pages/departments/community/480/العلاج-بالعسل/

ثانياً ليست الجبال بيوت النحل فقط فهو غالباً ما يعيش بالمناطق السهليّة الغنيّة بالأزهار و يعيش بخلايا خاصة من صنع الانسان و العاملات منه فيه من تقوم بصناعته !!

الرد :

يهدف صاحب الافتراء الطعن في القرآن  الكريم والاسلام ولكنه دون ان يقصد اثار انتباهنا الى اعجاز علمي آخر في القرآن الكريم !

صحيح ان النحل يستخدم رحيق الزهور nectar لصنع العسل ولكن الحقائق العلمية تقول بأن النحل يستطيب الثمار ويصنع منها العسل أيضاً ، مربو النحل يشهدون ان النحلات العاملة تستعمل شتى أصناف الفواكه من أجل صناعة العسل منها : الخوخ ،التفاح ،العنب  البرقوق ، الإجاص ( الكمثرى ) والتين .

وذلك بالضبط مثلما تقول الآية الكريمة

( ثُمَّ كُلِي مِنْ كُلِّ الثَّمَرَاتِ) النحل 69

إليكم المصدر من موقع يختص بتربية النحل




إضافة الى ذلك ، صاحب الافتراء يعترض على الآية ( يخرج من بطونها ) وبنفس الوقت لا يكمل الآية حتى لا يتضح الاعجاز العلمي فالآية الكريمة لم تقل ان (العسل ) يخرج من بطن النحل وانما ذكرت كلمة ( شراب) وهو مطابق للحقائق العلمية فالنحلات العاملات بعدما تجمع الرحيق من الأزهار والثمار وتهضمه داخل معدتها تفرزه من داخل بطنها عبر فمها سائلاً ويحتوى سائل الرحيق على اكثر من 70% -80٪ من الماء! والمصدر الذي استشهد به صاحب الافتراء يذكر هذه الحقائق واليكم المصدر مرة ثانيه


الغريب العجيب ان صاحب الكذبة  يفترض ان المسلمين يزعمون ان القران هو اول من ذكر ان للعسل فوائد طبيه ! مع ان الآية الكريمة لا تقول ذلك وانما تدعو الناس الى تأمل بديع صنع  الله في النحل وان العسل فيه فوائد طبيه

(فِيهِ شِفَاءٌ لِلنَّاسِ إِنَّ فِي ذَلِكَ لَآَيَةً لِقَوْمٍ يَتَفَكَّرُونَ) النحل 69

لاحظوا تلاعب اعداء الاسلام بالكلمات ودس الافتراضات بين ثنايا السطور !



ثالثاً : يستمر صاحب الافتراء بالتدليس ويقول " ليست الجبال بيوت النحل فقط" ! والرد هو كيف فهم ان الآية حصرت سكن النحل فقط في الجبال ! الم تذكر الآية ان هناك مساكن اخرى للنحل مثل الشجر ( في الجبال والسهول ) وبيوت البشر في كل مكان!

فالآية واضحه ولا تحتاج الى فطحل  في اللغة العربية لفهمها !

( أَنِ اتَّخِذِي مِنَ الْجِبَالِ بُيُوتًا وَمِنَ الشَّجَرِ وَمِمَّا يَعْرِشُونَ) النحل 68

- الافتراء:

الآية الثانيّة :

وَإِنَّ لَكُمْ فِي الْأَنْعَامِ لَعِبْرَةً ۖ نُسْقِيكُمْ مِمَّا فِي بُطُونِهِ مِنْ بَيْنِ فَرْثٍ وَدَمٍ لَبَنًا خَالِصًا سَائِغًا لِلشَّارِبِينَ ﴿-;---;--سورة النحل : 66﴾-;---;--

الفرث أو الغائط موجود في المثانة داخل البطن في حين أن الغدد الثدية حيث يتم إنتاج الحليب فيها تقع خارج البطن و الجسم عند الأنعام ( الماعز و البقر والابل ..) و بارزة في منطقة الصدر الأماميّة بجميع الثديّات لا بين الفرث و الدم يا رسول الله !!!

-

الرد :



يبدو ان آفة فهم اللغة العربية ما زالت تلازم المفتري على القرآن الكريم !

الآية لا تقول ان الحليب يخرج (من )( الفرث ) وهو الطعام المهضوم المتبقي في الكرش وليس في المثانة وكذلك لا تقول الآية الكريمة ان الحليب يخرج (من ) الدم!

بل تقول ان الحليب يخرج ( من بين ) فرث ودم. اي ان عملية انتاج الحليب لها علاقه مباشره مع (الفرث ) ومع (الدم ) وذلك مطابق تماماً للحقائق العلمية،

فالفرث او محتوى الكرش ضروري لإنتاج الحليب حيث تمتص المعده والأمعاء من (الفرث) الحوامض الأمينية ، الڤيتامينات ، حوامض الدهن والماء ومواد أخرى من المحتوى المهضوم وتنقلها الى الدورة الدموية . ثم يصل الدم الى ضرع البقرة المتكون من 4 غدد كبيره تنتهي بالحلمات ، تحتوي كل غده على شبكة اوعيه دموية وملايين الحويصلات alveoli الشبيهة بتلك الموجودة داخل الرئة ، تحتوي الحويصلات على خلايا متخصصة تأخذ المواد الغذائية من الدم وتصنع منها الحليب بتفاعلات معقده  وتأخذ من دم البقرة بروتينات مناعيه immunoglobulins اضافة للماء والڤيتامينات ، تتطلب عملية انتاج الحليب كميات كبيره من (الدم ) فمن أجل صنع ليتراً واحداً من الحليب يلزم تمرير اكثر من 400 ليتراً من الدم في ضرع البقرة.

لاحظوا الدور الحاسم للفرث والدم في انتاج الحليب ولاحظوا تطابق ذلك مع الحقائق العلمية وصدق الله العظيم منزل القرآن وباعث رسول الهدى بالحق



الافتراء:

يقول : "خُلِقَ مِنْ مَاءٍ دَافِقٍ * يَخْرُجُ مِنْ بَيْنِ الصُّلْبِ وَالتَّرَائِبِ / الطارق: 6 - 7

شرح المفسرين :

http://fatwa.islamweb.net/fatwa/index.php?page=showfatwa&Option=FatwaId&Id=38118

الإنسان لا يخلق من ماء المرآة ومن المعروف طبيّاً أن اتحّاد البويضة داخل الرحم مع نطفة واحدة من الرجل هو من يكوّن الجنين

ثانياً ذلك الماء لا يتكوّن من المنطقة الصدرية عندها ( الترائب ) , و لا يتكون مني الرجل من منطقته الصدريّة أيضاً ( الصلب ) هو يتكون في الخصيتين خارج البطن بعيداً عن منطقة الصدر !!

وهذا ايضاً حديث صحيح يوضح مقصد الآية اكثر

:" ماء الرجل أبيض وماء المرأة أصفر، فإذا اجتمعا فعلا مني الرجل مني المرأة أذكرا بإذن الله، وإذا علا مني المرأة مني الرجل أنثا بإذن الله " صحيح مسلم

http://fatwa.islamweb.net/fatwa/index.php?page=showfatwa&Option=FatwaId&Id=77653

ماء المرآة فهو فقط لترطيب المهبل لكي يسهل دخول عضو الرجل فيه ، هذا الماء يتكون من بويضات وسائل , السائل : يخرج من جميع خلايا الرحم وهو نفسه الغشاء المبطن للرحم - و أمّا البويضات تخرج من المبايض الأنثويّة وكلّما زادت الشهوه لدى المراه كلّما زادت افرازاتها ..



الرد :



هذه الآية الكريمة عباره عن اعجاز علمي كشفه العلم الحديث بعد اكثر من 1400 عام من نزول كلام الحق.

بدايةً : الرابط الاول الذي وضعه صاحب الشبهة ينسف افتراءه وكذبه لان الرابط يوضح ان المقصود ب( الماء الدافق ) هو المني المقذوف والايه الكريمة تقول أن الماء الدافق يخرج (من بين ) وليس ( من ) الصلب والترائب ! وفِي ذلك فرق كبير جداً ، فالآية لا تعني ان المني المقذوف يخرج من عظام الظهر ( صلب ) وعظام الصدر ( ترائب ) .

وانما تعني ان عظام الظهر والصدر لها علاقه وتأثير على المني المقذوف وقت الجماع ،وهو ما ذكره المفسرون وأكده العلم الحديث .

فقد تبين ان العظم يؤثر على انتاج المني في الخصيتين عن طريق إفراز هورمون الاوستوكالسين osteocalcin

والذي بدوره يؤثر على انتاج الهورمون الذكري "توستسترون " اللازم لإنتاج المني في الخصيتين واليكم مصادر علميه تثبت ذلك











إضافةً الى ذلك ، فقد أسفرت الابحاث عن الكشف عن علاقة عظام الظهر ( الصلب ) وأضلاع الصدر ( الترائب ) بقذف المني اثناء الجماع الجنسي ، ذلك لأنه توجد في الحبل الشوكي الموجود داخل فقرات الظهر ( الصلب ) بؤر عصبيه مهمتها ضبط وتسيير عملية قذف المني أثناء ممارسة الجنس ، وتُسمى تجمعات الاعصاب تلك ب " مولد القذف الشوكي " " Spinal Ejaculation Generator وهي مسؤوله عن ضبط المعلومات الحسيه والحركية القادمة من الدماغ ومن اعصاب منطقه الحوض وإحداث عملية قذف المني عن طريق تحفيز " العصب الخثلي " hypogastric ganglion

وهو عصب ينزل من الفقرة الصدرية العاشرة T10 ( لاحظوا العلاقة مع الترائب) الى الفقرة القطنية الثانية وهناك يتفرع ويتخذ طريقه حتى الخصيتين



واليكم مصدراً علمياً


وللتلخيص : العظم يؤثر على انتاج المني وكذلك داخل فقرات الظهر ( الصلب )تتواجد مراكز عصبيه تتحكم وتضبط عملية قذف المني بالاشتراك مع عصب مصدره من أضلاع الصدر ( الترائب ) .

بالنسبه للقسم الثاني من الشبهة ، فلم يقل القران الكريم ولا الحديث الشريف ان الانسان يُخْلق فقط من ماء الرجل ! وكذلك الحديث المذكور في الشبهة اعلاه هو بحد ذاته اعجاز علمي في السنه النبوية! فعندما نقرأه "ماء الرجل أبيض وماء المرأة أصفر، فإذا اجتمعا فعلا مني الرجل مني المرأة أذكرا بإذن الله، وإذا علا مني المرأة مني الرجل أنثا بإذن الله " صحيح مسلم

نرى ان الرسول صلى الله عليه سلم ذكر كلمة " علا " ومن معاني كلمة علا : الغلبة والتفوق .

وفِي عصرنا هذا أكد العلم الحديث ان جنس المولود يتقرر بحسب كروموزوم الجنس الذي ينقله الحيوان المنوي فاذا كان الكروموزوم من نوع x يكون الجنين أنثى اما اذا كان الكروموزوم من نوع y يكون جنس الجنين ذكراً ، بالنسبة للبويضة الموجودة في سائل المرأة فدائماً تحتوي على كرومزم x .



بكلمات اخرى اذا وافق الحيوان المنوي البويضه من ناحيه نوع الكرموزم (x) فسيكون المولود أنثى اما اذَا خالفها وكان نوع الكروموزوم (y) فعندئذٍ يتغلب كروموزوم (y)على (x) الأنثوي الجنس ،ويكون جنس المولود ذكراً .

كما ترون القران الكريم والسنة النبوية

يطابقان المكتشفات العلمية ويوافقانهم

والحمد لله على نعمة الاسلام



الافتراء:

4 - الجبال في القرآن:

الجبال في القرآن وضعت حتى تحفظ توازن الأرض ولا تميد، وفي هذا أيضا اشارة الى أن الأرض ثابتة في القرآن، يقول:

وَأَلقَى فِي الأَرضِ رَوَاسِيَ أَن تَمِيدَ بِكُم وَأَنهَارًا وَسُبُلًا لَعَلَّكُم تَهتَدُونَ

وَجَعَلنَا فِي الأَرضِ رَوَاسِيَ أَن تَمِيدَ بِهِم وَجَعَلنَا فِيهَا فِجَاجًا سُبُلًا لَعَلَّهُم يَهتَدُونَ

الأرض لا تحيد عن مجال دورانها الثابت بفعل قوى الجاذبية بين الكواكب و النجوم المتجاورة لا علاقة للجبال على الأرض أو منخفضاتها بأي دور في استقرار حركتها

الرد :

الآيات الكريمة اعلاه هي اعجاز علمي كشفه علم طبقات الارض ليس اقل من ١٣ قرناً منذ نزول القرآن الكريم . يتكون سطح الكره الأرضية crust من 8 صفائح او الواح ضخمه وعشرات الصفائح الصغيره tectonic plates وهي تحمل اليابسة والمحيطات وتطفو كل الصفائح فوق الطبقة الثانية التي تليها وهي طبقه (الوشاح ) mantle وهي طبقه لزجه وذات درجة حراره اعلى من قشرة الكره الارضيه . للمزيد من المعلومات




القسم الظاهر من الجبال فوق سطح الارض عباره عن جزء صغير من حجمها الكلي ذلك لان جذور الجبال تصل الى ٥-١٠ أضعاف الجزء البارز فوق سطح الارض وتنغرس جذور الجبال عميقاً في الطبقة الثانية ( الوشاح ) تتوازن الجبال حسب قوانين طفو الأجسام المغمورة جزيئاً داخل السوائل وكي يتحقق ذلك يجب ان يكون اغلب حجمها مغموراً في السائل او طبقة الوشاح وتسمى هذه الخاصية في علم الجيولوجيا ب التوازن الثقلي Isostasy . للمزيد من القراءة


انغراس جذور الجبال يثبت صفائح قشرة الكره الأرضية ويمنعها من الميلان والتأرجح كلوحة خشب طافيه على الماء ، لاحظوا ان القران الكريم أشار الى ان الجبال مغموره في سائل اذ تذكر الآيات الكريمة كلمة " رواسي " لتشير الى حقيقة كوّن أسفل الجبال مستقراً في وسط سائل لزج !

هذا الفيلم القصير يوضح لكم الاعجاز بصوره موجزه



كما ترون أعزاءي ؛ فالآيات عباره عن اعجاز علمي مذهل وصاحب الافتراءات اما بدافع الجهل او الحقد او كليهما يخلط بين تثبيت الجبال لسطح الكره الأرضية وبين دوران الكره الأرضية حول الشمس !





الافتراء:

5 - ثبوت الأرض الخاطىء بالقرآن

إِنَّ اللَّهَ يُمْسِكُ السَّمَاوَاتِ وَالأَرْضَ أَنْ تَزُولا وَلَئِنْ زَالَتَا إِنْ أَمْسَكَهُمَا مِنْ أَحَدٍ مِنْ بَعْدِهِ " / فاطر: 41

أَمَّنْ جَعَلَ الْأَرْضَ قَرَاراً – سورة النمل : 61

اللَّهُ الَّذِي جَعَلَ لَكُمُ الْأَرْضَ قَرَاراً وَالسَّمَاءَ بِنَاءً – غافر الآية: 64 / النمل : 61 - اي :جعل الأرض مستقرّةً بالدّحو و التـّسوية , و القرار هو الثبوت وعدم الحركة .

رب المشارق والمغارب " / المعارج:40

وقد ورد في بيان الأصول التي اجتمع عليه أهل السنة والجماعة ص 330 طبعة دار المعرفة ببيروت :

" ان القول بالدوران ينافي ما اتفق عليه الناس من أن هذه الأرض منها بلاد مشرقية ومنها بلاد مغربية وهذا يتعارض مع القول بالدوران لأنه على هذا القول كل بقعة من الأرض مشرق ومغرب فعليه لا بلاد مشرقية ولا بلاد مغربية " و بحديث صحيح عن عبدالله بن عمر قال ص “ لا تجمع أمتي على ضلالة ”

https://m.facebook.com/note.php?note_id=384779838253530

http://a.islamreligion.fr/thoubout-ard-dawaran-chams/

نفى القرآن تأثير أي حركة للأرض في حدوث الظل

أَلَمْ تَرَ إِلَى رَبِّكَ كَيْفَ مَدَّ الظِّلَّ وَلَوْ شَاءَ لَجَعَلَهُ سَاكِنًا ثُمَّ جَعَلْنَا الشَّمْسَ عَلَيْهِ دَلِيلًا الفرقان : 45

نقض محاولات اثبات الدوران المحرّفة لمعاني آيات القرآن الحقيقيّة

الآيّة "وترى الجبال تحسبها جامدة وهي تمر مر السحاب" التي يتشدق بها بعض دعاة الاعجاز على انها دليل لدوران الأرض حول نفسها هي تخالف آيات وأحاديث ثبوت الأرض وتناقض الإثباتات العلميّة أيضاً فالسحب لا تتحرك نتيجة دوران الارض ولا تتحرك وتدور اصلا كما تتحرك الارض و هي تتحرك بشكل عشوائي حسب حركة جزيئات الهواء التي تتولّد بسبب اختلاف فروقات الضغط الجوي في كل منطقة .

أمّا معنى وسبب منشأ هذه الآية البسيط هو انه هذا ما رآه ذاك المدّعي عندما وقف على سفح جبل فرآها وكأنها تتحرك عبر السحاب

أما بالنسبة لآية " يكور الليل على النهار" فهي حسب معجم اللغة و شروحات و تفسيرات جميع مفسري القرآن تعني " أَدخل هذا في هذا " و اذا فرضاً يقصد بها التكوير هنا .. فهل يمكن تكوير الليل أو النهار و هل يمكن تكوير فترة زمنيّة !! هذا الأمر لا تقبله اللغة و لا حتّى أساسيات للمنطق .. "

الرد :

لم يذكر القرآن بتاتًا ان الارض ثابته لا تتحرك ! كل ما ذُكرَ أعلاه عباره عن افتراضات مضلله يزج بها الحاقد على الاسلام بهدف التشويش لا غير !



"إِنَّ اللَّهَ يُمْسِكُ السَّمَاوَاتِ وَالأَرْضَ أَنْ تَزُولا وَلَئِنْ زَالَتَا إِنْ أَمْسَكَهُمَا مِنْ أَحَدٍ مِنْ بَعْدِهِ " (فاطر: 41)

معنى الآية الكريمة أن فقط العناية الالهية تتسبب في استمرار تمدد الكون( السماوات )وعدم انهياره. فكما هو معروف علميًا, نشأ الكون بسبب حدوث الانفجار العظيم تلاه تكون الطاقة والمادة وبعد ذلك تكونت النجوم والمجرات ، ومن الثابت علميًا أن الكون في تمددٍ مستمر وسبب التمدد هو تغلب طاقة التمدد او الدفع والتي تسمى علميًا بالطاقة السوداء Dark energy على قوة الجذب التي تعارض التمدد وللمزيد اقرأوا هنا


لا أحد يعلم تمامًا ماهية الطاقة السوداء ولا حتى مُسَّبب قوة الجذب

والآية الكريمة تعني ان من يمنع انهيار السماوات على نفسها وانسحاقها وبضمنها كرتنا الأرضية هو فقط رب العالمين ولا أحد غيره ! فهو المتحكم بالطاقة السوداء وهو الضابط لمقدار قوة الجاذبية .



بالنسبة للآيتين الكريمتين :

أَمَّن جَعَلَ الْأَرْضَ قَرَارًا وَجَعَلَ خِلَالَهَا أَنْهَارًا وَجَعَلَ لَهَا رَوَاسِيَ وَجَعَلَ بَيْنَ الْبَحْرَيْنِ حَاجِزًا ۗ أَإِلَٰهٌ مَّعَ اللَّهِ ۚ بَلْ أَكْثَرُهُمْ لَا يَعْلَمُونَ (النمل 61)

اللَّهُ الَّذِي جَعَلَ لَكُمُ الْأَرْضَ قَرَارًا وَالسَّمَاءَ بِنَاءً وَصَوَّرَكُمْ فَأَحْسَنَ صُوَرَكُمْ وَرَزَقَكُم مِّنَ الطَّيِّبَاتِ ۚ ذَٰلِكُمُ اللَّهُ رَبُّكُمْ ۖ فَتَبَارَكَ اللَّهُ رَبُّ الْعَالَمِينَ ( غافر 64)

فلا يوجد فيهما ذِكْر لثبوت الارض في الفضاء ! بل تتحدثان بوضوح تام عن ان الارض عباره عن (قرار ) يعني مكان يستقر فيه الانسان ويسكنه ، وهذا ما تذكره كتب التفسير واللغة

(قَرَارًا : تستقرون عليها, وتسكنون فوقها)


لاحظوا كيف يخلط المدلس ويتلاعب بمعاني الكلمات !

بالنسبة للآية الكريمة :

أَلَمْ تَرَ إِلَىٰ رَبِّكَ كَيْفَ مَدَّ الظِّلَّ وَلَوْ شَاءَ لَجَعَلَهُ سَاكِنًا ثُمَّ جَعَلْنَا الشَّمْسَ عَلَيْهِ دَلِيلًا (الفرقان 45)

فجنبًا الى روعتها البلاغية والإنشائية تذكر الآية ان (الشمس ) هي الدليل على الظل وليس ( الارض ) !

لا أعلم من أين استنتج المفتري ان القران الكريم ينكر دوران الكرة الارضية حول الشمس !؟ أو أن دوران الارض حول نفسها لا يساهم في حدوث الظل ؟ فالآية تتحدث بوضوح عن مد الظل وقبضه وذلك لا يحدث الا بدوران الارض حول نفسها وحول الشمس !

لم يقل القرآن الكريم ولا السنة النبوية الشريفة ان الأرض ثابتة لا تتحرك. هذه ادعاءات باطلة وكاذبه وبالنسبة للآية الكريمة

 (وَتَرَى الْجِبَالَ تَحْسَبُهَا جَامِدَةً وَهِيَ تَمُرُّ مَرَّ السَّحَابِ ۚ صُنْعَ اللَّهِ الَّذِي أَتْقَنَ كُلَّ شَيْءٍ ۚ إِنَّهُ خَبِيرٌ بِمَا تَفْعَلُونَ ( النمل 88)

فقد وردت ضمن آيات تتحدث عما سيحدث يوم الحساب بعد النفخ في الصور وليس في الحياة الدنيا ! وحتى يتضح المعنى أكثر اليكم كل الآيات:

" وَيَوْمَ يُنْفَخُ فِي الصُّورِ فَفَزِعَ مَنْ فِي السَّمَوَاتِ وَمَنْ فِي الْأَرْضِ إِلَّا مَنْ شَاءَ اللَّهُ وَكُلٌّ أَتَوْهُ دَاخِرِينَ (87) وَتَرَى الْجِبَالَ تَحْسَبُهَا جَامِدَةً وَهِيَ تَمُرُّ مَرَّ السَّحَابِ صُنْعَ اللَّهِ الَّذِي أَتْقَنَ كُلَّ شَيْءٍ إِنَّهُ خَبِيرٌ بِمَا تَفْعَلُونَ (88) مَنْ جَاءَ بِالْحَسَنَةِ فَلَهُ خَيْرٌ مِنْهَا وَهُمْ مِنْ فَزَعٍ يَوْمَئِذٍ آَمِنُونَ (89) وَمَنْ جَاءَ بِالسَّيِّئَةِ فَكُبَّتْ وُجُوهُهُمْ فِي النَّارِ هَلْ تُجْزَوْنَ إِلَّا مَا كُنْتُمْ تَعْمَلُونَ (90)"

الحمد لله على نعمة الايمان ونعمة القرآن ، ومن يدَعي ان القرآن الكريم من صنع البشر, فليتفضل وليثبت لنا ذلك وليؤلف لنا بكلماته شيئًا مثل القرآن الكريم وليضع فيه من أخبار الغيب والسالفين وليضمنه اعجاز علمي ولغوي !!

الافتراء:

6 - طبقات الأرض و السماء السبعة مغلوطة علميّاً !!!

و التي توافق كلام الكتب الدبنيّة الأخرى ما قبل الاسلام و هو الاعتقاد السائد قديماً :

طبقات الأرض الثلاثة ( من المواقع المختصّة ) :

http://geology.com/nsta/earth-internal-structure.shtml

http://science.nationalgeographic.com/science/earth/inside-the-earth

http://en.wikipedia.org/wiki/Structure_of_the_Earth

طبقات السماء الأرضيّة الخمسة فقط ( من موقع وكالة ناسا )

http://airs.jpl.nasa.gov/maps/satellite_feed/atmosphere_layers/





الرد :

صاحب الافتراء يرمي بالأكاذيب مباشرة من غير ان يذكر عن آيات يتحدث ، على أية حال ذكر القران الكريم وجود سبع سموات فوق بعضها البعض ، اليكم بعضًا من الآيات الكريمة:

الَّذِي خَلَقَ سَبْعَ سَمَاوَاتٍ طِبَاقًا ۖ ﴿٣ الملك﴾

تُسَبِّحُ لَهُ السَّمَاوَاتُ السَّبْعُ وَالْأَرْضُ وَمَنْ فِيهِنَّ ﴿٤٤ الإسراء﴾

قُلْ مَنْ رَبُّ السَّمَاوَاتِ السَّبْعِ وَرَبُّ الْعَرْشِ الْعَظِيمِ ﴿٨٦ المؤمنون﴾

ثُمَّ اسْتَوَىٰ إِلَى السَّمَاءِ فَسَوَّاهُنَّ سَبْعَ سَمَاوَاتٍ ﴿٢٩ البقرة﴾



وكرتنا الأرضية عباره عن ذرة ضمن مليارات الكواكب في السماء الأولى والعلم عاجز عن الإحاطة بكل الكون المرئي وهي السماء الاولى, والعلم قاصر عن رصد كل السماء الاولى, فبحسب تقديرات العلماء فالكون (على الأقل ) اكبر ب 250 ضعفاً من حجم الكون المرصود من قبل العلماء


كما ترى ، عزيزي القارئ ، العلم يعجز عن الإحاطة بالسماء الأولى وذلك يجعل من الحديث عن السماء الثانية وباقي السموات حديث عن غيبيات لا يمكن فحصها علميًا بسبب قصور العلم وعجزه ، فكيف يمكن للمفتري على القرآن الكريم ان ينفي وجودهما بثقة تامه ! هل احاط بالسماء الاولى وخرج منها وتأكد من عدم وجود سموات أخرى !! ام ان الكراهية للإسلام تحركه وتدعوه الى الهذيان . نحن نؤمن بوجود سموات سبع لأن القرآن الكريم والسنة النبوية أخبرانا بذلك ولو كان العلم قادرًا على اختراق السماء الأولى لاكتشف ما ذكره القرآن الكريم بالضبط .

واذا كانت قضية وجود سبع سموات مذكورة في كتب دينيه قبل الإسلام فليس في ذلك غرابة ، الم يرسل الله رسله الى كل الشعوب والأمم ! الم تكن الرسالة تتكرر كل مرة، ولكن الناس مع مرور الزمن كفرت وحرفت الى أن نزل القرآن الكريم ليعيد ويثبت الصيغة الأصلية للأديان والرسالات ويكون هو الدين الأخير وكتابه المحفوظ من اي تحريف وتغيير !؟



بالنسبة لموضوع طبقات الأرض ، فبينما يحاول كاتب الافتراء التدليس على القرآن الكريم يقوم دون أن يقصد بالإشارة الى إعجازٍ علميٍ ذكره القرآن الكريم وأكدته علوم الجيولوجيا .

ذكر القران الكريم ان السموات هي سبع وأنها تغلف بعضها بعضًا مثل داخل حبة البصل

الَّذِي خَلَقَ سَبْعَ سَمَاوَاتٍ طِبَاقًا ۖ ﴿٣ الملك﴾

اقرأوا هنا تفسير الآية الكريمه


اضافةً الى ذلك فقد ذكر القرآن الكريم ان نفس النمط موجود في الأرض انظروا الآية الكريمة

"اللَّهُ الَّذِي خَلَقَ سَبْعَ سَمَاوَاتٍ وَمِنَ الْأَرْضِ مِثْلَهُنَّ يَتَنَزَّلُ الْأَمْرُ بَيْنَهُنَّ " (١٢ الطلاق )

من قراءة كلا الآيتين الكريمتين نستنتج أن القرآن الكريم قال أن هناك سبع طبقات أرضية ، وهو ما توصل اليه علماء طبقات الارض . كان تقسيم طبقات الارض في السابق تصنيفًا بسيطًا للغايه يعتمد على 3 طبقات : القشره الأرضية crust ، طبقة الوشاح mantle والنواة core ، بعد ذلك اكتشف العلماء ان طبقة الوشاح والنواه كل واحد منهما مبنية من طبقتين وشاح علوي وسفلي upper-lower mantle ونواة خارجيه وداخليه outer- inner core وبذلك صار عدد طبقات الارض 5 ، وهو ما توقف عنده صاحب الفرية كي يدعي ان في القرآن أخطاء علمية ولم يذكر ان أبحاث علماء الجيولوجيا أفضت الى اكتشاف وجود 4 طبقات مكوّنه لطبقة الوشاح

The mantle is divided into several layers: the upper mantle, the transition zone, the lower mantle, and D” (D double-prime), the strange region where the mantle meets the outer core.

تقسم طبقة الوشاح الى عدة طبقات ،الوشاح العلوي -ويحتوي على

( lithosphere و asthenosphere). المنطقة الانتقالية، الوشاح السفلي, والطبقة الرئيسية المزدوجة

(D double-prime)

للمزيد إقرأوا هنا :


إذاً فعلميًا عدد طبقات الأرض سبعة

١- القشرة الأرضية crust

٢- أربع طبقات الوشاح mantle

٣- طبقتي النواة core

وسبحان الله يأتي الحاقد على الإسلام ليثير الشكوك في القرآن الكريم ولكنه من حيث لا يقصد يتسبب بالتنبيه الى الاعجاز العلمي المكنون في كتاب الله عز وجل

وَاللَّهُ غَالِبٌ عَلَىٰ أَمْرِهِ وَلَٰكِنَّ أَكْثَرَ النَّاسِ لَا يَعْلَمُونَ (يوسف 21)





الافتراء:

7 - شكل السماء في القرآن:

السماء في القرآن عبارة عن بناء صلب مرفوع عن سطح الأرض ليس له شقوق ومزيّن بالنجوم، فالقرآن يقول:

خَلَقَ السَّمَاوَاتِ بِغَيرِ عَمَدٍ تَرَونَهَا وَأَلقَى فِي الأَرضِ رَوَاسِيَ أَن تَمِيدَ بِكُم ( لقمان:10 )

يَنظُرُوا إِلَى السَّمَاءِ فَوقَهُم كَيفَ بَنَينَاهَا وَزَيَّنَّاهَا وَمَا لَهَا مِن فُرُوجٍ ( ق:6 )

- وَلَقَدْ خَلَقْنَا السَّمَاوَاتِ وَالْأَرْضَ وَمَا بَيْنَهُمَا سورة ق 38

- هُوَ الَّذِي خَلَقَ لَكُمْ مَا فِي الْأَرْضِ جَمِيعًا ثُمَّ اسْتَوَى إِلَى السَّمَاءِ فَسَوَّاهُنَّ سَبْعَ سَمَاوَاتٍ . (البقرة :29)

السَّمَاوَاتِ وَالْأَرْضَ كَانَتَا رَتْقاً فَفَتَقْنَاهُمَا (ملتحمتان ففصلناهما) (الأنبياء: 30)

هذا الأمر من غير الممكن تصوره الا اذا كانت الأرض مسطّحة و السماء فوقها تناظرها ( وهذا ما يطابق تصوّرات بطليموس لشكل الكون ) حيث اعتبر ان النجوم و الكواكب مكوّنة من تجمّعات على سبع طبقات متناظرة في السماء وانها صلبة عدا السماء الدنيا و مرفوعة من دون عمد و الأرض ثابتة و هي اكبر من الكواكب و النجوم . و نجد هذا جليّا بتصوراته وآياته التي توضح ذلك " وَيُمْسِكُ السَّمَاءَ أَنْ تَقَعَ عَلَى الْأَرْضِ – سورة الحج 65" " رَفَعَ السَّمَاوَاتِ بِغَيْرِ عَمَدٍ - الرعد:2 " " وجعلنا السماء سقفا محفوظاً " الأنبياء :32 - وَالسَّمَاءَ رَفَعَهَا وَوَضَعَ المِيزَانَ الرحمن:7 - وَيُمسِكُ السَّمَاءَ أَن تَقَعَ عَلَى الأَرضِ إِلَّا بِإِذنِهِ - وبنينا فوقكم سبعا شدادا

فهل يمكن تصوّر أن الأرض انفصلت عن الفضاء و مليارات المجرات الكونية و التي تحوي مليارات المليارات من الكواكب و النجوم و التي تحاوطها من كل الجهات -وهل يمكن فصل الفضاء عن الأرض التي هي مفصولة عنها بالأساس ؟؟؟ وهل يمكن شقّ السماء ان لم تكن هي صلبة بالأساس " اذا السماء انشقت " إِذَا السَّمَاءُ انفَطَرَت - وفـُتحت السماء فكانت ابوابا " ..

فشكل الأرض و الفضاء كان وفق نظريات الفلكيين القدماء كبطليموس و غيره

فيما لم يرد ايّة اية واحدة بها تثبت تكوير الأرض بشكل مبيّن فكانت دوماً مسطّحة !!: ""

الرد :

كل الادعاءات المذكورة أعلاه عبارة عن اكاذيب وافتراءات ولا علاقة لها بآيات القرآن الكريم.

آيات القرآن لا تقول بان الارض ثابتة او انها مسطحه ! ولا أدري كيف ربط المستلحد بين كوّن السماء مرفوعة بلا عمد وبين افتراض سطحية وثبات الارض !

فالسماء تحيط بالكرة الأرضية من كل جانب، وآيات القرآن الكريم تدعو البشر بان يتأملوا السماء والتي دائما تبدو سقفًا فوق سطح الكرة الأرضية . ونَعَمْ مادة السماء ليس فيها شقوق او خروق ومع ذلك لم يقل القرآن الكريم ان السماء عبارة عن مبنى صلب .

بل ان اياته كشفت قبل اكثر من ١٤ قرنًا ان السماء ذات مبنى محبوك .

"وَالسَّمَاءِ ذَاتِ الْحُبُكِ "(الذاريات 7)

وفعلًا ؛ اكتشف علماء الفضاء ان مُرَكبات مادة الكون ، عبارة عّن 4.6% من جزيئات المادة ( نويترونات ، بروتونات وكذا ..) ، اما البقيه 95.4% فمُكونة من المادة والطاقة السوداء . والامر الاكثر اذهالًا هو اكتشاف وجود الطاقة السوداء في شبكات معقدة متشابكه من الخيوط الكونية ، وتقوم خيوط المادة الكونية بالربط بين ملايين المجرات

We now have our first solid piece of evidence that this matter has been hiding in the delicate threads of cosmic webbing bridging neighboring galaxies

Discoveries in recent years that have established vast webs of dark matter stretching across

space



للمزيد من القراءة ، اقرأوا هنا












والحقيقة أن مفهوم السماء في القرآن الكريم سبق العلوم بقرون عديدة. ومن ذلك ان السماء موجودة في حالة تمدد واتساع، وهي حقيقة علمية تم التوصل اليها فقط في نهاية العشرينيات من القرن العشرين ،على يد عالم الفضاء " ادوين هابل ". وللمزيد اقرأوا هنا


من جهة ثانية ، تأملوا الآية الكريمة التالية وتبينوا بأنفسكم كيف ان القرآن الكريم يزخر بإشارات كونية معجزة ، فقد ورد ذكر هذة الحقيقة في سورة الذاريات !

وَالسَّمَاءَ بَنَيْنَاهَا بِأَيْدٍ وَإِنَّا لَمُوسِعُونَ (الذاريات 47)

بالنسبة للادعاء القائل بان معلومات القرآن منقولة عن تصور " بطليموس " . فهذا افتراء محض وكذب سافر،

وذلك لان بطليموس اعتقد ان الارض ثابتة لا تدور وأنها مسطحة، وأعتقد" بطليموس " ان الأرض هي مركز الكون، وان القمر وباقي الأجرام السماوية تدور حول الأرض في افلاك دائرية كاملة الاستدارة. وكل كوكب او جرم موجود داخل كرة sphere . وللمزيد عن افكار " بطليموس " اقرأوا هنا


واضح جدًا ، عزيزي القارئ ان تصورات " بطليموس " بعيدة جدًا عن فحوى آيات القرآن الكريم . وان بطليموس لم يتحدث عن سبع سموات مثلما يَزعُم صاحب الافتراء والاكاذيب !

ومرة اخرى أتحدى كل من يحاول التشكيك في مصدر القرآن الكريم، أن تفضل وألف لنا من كلماتك شيئًا مثل القرآن الكريم ! اكتب لنا نصًا فيه ؛اعجاز علمي لم يُكتشف بعد ، فيه أخبار عن عالم الغيب لم يسمع بها احد واكتب ذلك بأسلوب بلاغي لا يمكن تقليده !

أما فيما يتعلق باتصال السماء مع الارض . فذلك أيضًا اعجاز علمي ، يحاول الحاقدون تشويهه للطعن في القرآن الكريم .

فالآية الكريمة استعلمت كلمة " رتقًا "

أَوَلَمْ يَرَ الَّذِينَ كَفَرُوا أَنَّ السَّمَاوَاتِ وَالْأَرْضَ كَانَتَا رَتْقًا فَفَتَقْنَاهُمَا ۖ وَجَعَلْنَا مِنَ الْمَاءِ كُلَّ شَيْءٍ حَيٍّ ۖ أَفَلَا يُؤْمِنُونَ

 (الأنبياء 30)

والمعنى اللغوي لكلمة " رَتْقْ" هو الشيء المُلتحم المسدود؛ اي أن الآية الكريمة تقول ان السموات والارض كانت كلها مادة واحدة، جسمًا واحدًا . وليس معنى الآية الكريمة ان الارض كانت مخلوقة جاهزة وملتصقة بالسماء ثم انفصلت عنه ! فذلك تدليس من المفتري وتشويش .



وتخبرنا آيات القرآن الكريم ان السماء كانت قبل خلق الارض وبعدها، عبارة عن دخان



قُلْ أَئِنَّكُمْ لَتَكْفُرُونَ بِالَّذِي خَلَقَ الْأَرْضَ فِي يَوْمَيْنِ وَتَجْعَلُونَ لَهُ أَندَادًا ۚ ذَٰلِكَ رَبُّ الْعَالَمِينَ (9) وَجَعَلَ فِيهَا رَوَاسِيَ مِن فَوْقِهَا وَبَارَكَ فِيهَا وَقَدَّرَ فِيهَا أَقْوَاتَهَا فِي أَرْبَعَةِ أَيَّامٍ سَوَاءً لِّلسَّائِلِينَ (10) ثُمَّ اسْتَوَىٰ إِلَى السَّمَاءِ وَهِيَ دُخَانٌ فَقَالَ لَهَا وَلِلْأَرْضِ ائْتِيَا طَوْعًا أَوْ كَرْهًا قَالَتَا أَتَيْنَا طَائِعِينَ (11) سورة فصلت

وحقيقة كوّن السماء دخان، هي شيء يقول به العلم ، فالسماء كانت عبارة عن غيمة ضخمة من غبار جزيئات المادة والتي تكونت بعد مليارات السنين من نشأة الكون - الانفجار العظيم . وبحسب المستجدات العلمية ، تقريبًا قبل 4.6 مليار سنة انفصلت ( حدوث عملية الفَتْقْ ) غيمة من سحابة الغازات الضخمة تسمى تلك الغيمة ب( السديم الشمسي ) solar nebula وتكتل محتواها بفعل قوة الجاذبية الى الشمس وباقي كواكب المجموعة الشمسية .

للمزيد من المعلومات ، اليكم هذا الرابط




مرة أخرى ودائما, نلاحظ إعجاز القرآن العلمي وتطابق آياته مع الحقائق العلمية المكتشفة.



بالنسبة لمسألة سطحية الأرض ، فآيات القرآن الكريم تشير الى كروية الأرض وليس الى سطحيتها. لنتمعن آيات القرآن الكريم بصدد هذا الموضوع؛



أَفَلَا يَنظُرُونَ إِلَى الْإِبِلِ كَيْفَ خُلِقَتْ (17) وَإِلَى السَّمَاءِ كَيْفَ رُفِعَتْ (18) وَإِلَى الْجِبَالِ كَيْفَ نُصِبَتْ (19) وَإِلَى الْأَرْضِ كَيْفَ سُطِحَتْ (20) سورة الغاشية

تخاطب الايات الكريمة ، الناس على وجه الأرض وتدعوهم الى تأمل إبداع الخالق، ومن ذلك حُسن خِلقَة الإبل وكيف أن السماء مرفوعة وأن الجبال منصوبة ، وكيف أن الأرض مسطحةً.

وطبعًا دائما تبدو الأرض مسطحة لمن يتأملها وهو واقف على سطحها !

بل إن الآية صحيحة دائمًا فقط في حالة كروية الارض، فلو كانت الأرض مسطحه شكلًا لما كانت الآية صحيحة لمن يقف قبل حافة الأرض بقليل وينظر أمامه ، لأنه سيراها مبتورة غير مسطحةً !

وَالْأَرْضَ مَدَدْنَاهَا وَأَلْقَيْنَا فِيهَا رَوَاسِيَ وَأَنبَتْنَا فِيهَا مِن كُلِّ شَيْءٍ مَّوْزُونٍ (19) سورة الحجر

تفيد الآية الكريمة أن الأرض ممتدة ، والارض تبدو دائمًا ممتدة لمن يقف عليها، فقط في حالة الشكل الكروي وليس المسطح !

خَلَقَ السَّمَاوَاتِ وَالْأَرْضَ بِالْحَقِّ ۖ يُكَوِّرُ اللَّيْلَ عَلَى النَّهَارِ وَيُكَوِّرُ النَّهَارَ عَلَى اللَّيْلِ ۖ وَسَخَّرَ الشَّمْسَ وَالْقَمَرَ ۖ كُلٌّ يَجْرِي لِأَجَلٍ مُّسَمًّى ۗ أَلَا هُوَ الْعَزِيزُ الْغَفَّارُ (5) سورة الزمر

تستعمل الآية الكريمة كلمة ( يُكَوِّرُ ) لوصف تعاقب الليل على النهار والنهار على الليل . و ( يُكَوِّرُ ) مشتقة من (كُرَة) وفِي ذلك إشارة واضحة الى ان حالة الليل والنهار على سطح الأرض ، تتخذ شكلًا كرويَّا !



.



فسبحان الله والحمد لله



الافتراء:

8 - الأرض مبسوطة ومسطّحة بالقرآن

أَفَلاَ يَنظُرُونَ .. إِلَى ٱ-;---;--لأَرْضِ كَيْفَ سُطِحَتْ / الغاشية:20

وَإِذَا الْأَرْضُ مُدَّتْ / الانشقاق: 3

والله جعل لكم الأرض بساطا / سورة نوح :19

أَوَلَمْ يَرَوْاْ أَنَّا نَأْتِي الأَرْضَ نَنقُصُهَا مِنْ أَطْرَافِهَا الرعد/41 .و الأنبياء/44

وَالْأَرْضَ بَعْدَ ذَلِكَ دَحَاهَا / النازعات :30

وَالْأَرْضَ مَدَدْنَاهَا وَأَلْقَيْنَا فِيهَا رَوَاسِيَ وَأَنْبَتْنَا فِيهَا مِنْ كُلِّ شَيْءٍ مَوْزُونٍ / الحجر: 19 /قوله مددناها وجعلنا فيها رواسي والأرض بعد ذلك دحاها لا يعني البتة كيف تنظر العين البشريّة لها كما يدّع بعض الشيوخ في محاولة الهروب من قضية بسط الأرض بالقرآن

والأرض وما طحاها " سورة الشمس : 6 . قال مجاهد ، وقتادة والضحاك ، والسدي ، والثوري ، وأبو صالح ، وابن زيد : ( طحاها ) اي دحاها أو بسطها . وهذا أشهر الأقوال ، وعليه الأكثر من المفسرين ، وهو المعروف عند أهل اللغة ، قال الجوهري : طحوته مثل دحوته ، أي : بسطته .

كلمة دحا تعرّضت للكثير من التحريف و التشويه من قبل بعض المحرّفين عن طريق تبديلها بمعنى الأدحيّة والتي تعني مكان مبيض النعام والذي هو بشكل مسطّح أو مقعّر قليلاً ( قطع زائد ) , و لا علاقة له البتّة بالبيضة ولا بشكلها ,

و التي اصلاً لغويّاً خاطئة فلا يمكن التشبيه بدون اداة للتشبيه ( / كـ / الكاف - كأنّ - مثل ..... ) وهي اقرب لشكل الكرة من ان تكون كبيضة النعام !!! ..

دحاها او مدّت ليس لهما الّا معنى واحد و هو البسط او التسطيح بمعاجم اللغة العربية

http://www.maajim.com/دحا

http://www.baheth.info/web/all.jsp?term=دحا

http://www.almaany.com/home.php?language=arabic&lang_name=عربي&word=دحا

عدا معجم اللغة العربية المعاصر و مؤلّفه الداعية الإسلامي أحمد مختار عمر الذي خالف جميع معاجم اللغة العربيّة و تلاعب بمفرداتها وحرّفها ليثبت صدق دجل مؤلّف القرآن !!

والذي أصدر سنة 2008 م !!

http://www.waqfeya.com/book.php?bid=5544

- الرد على مدلسين القرآن و مدّعين الاعجاز العلمي

 الآية : " يُكَوِّرُ اللَّيْلَ عَلَى النَّهَارِ وَيُكَوِّرُ النَّهَارَ عَلَى اللَّيْلِ " لا تدل على تكوير الأرض مطلقاً بل أنها تعني بالحقيقة وحسب اللغة و ما يقتضيه المنطق : يدخل الليل على النهار و العكس أيضاً و كُورت:اي غّوِّرت، أُذهِب ضوؤها. و كورت الشمس: جمع ضوؤها و لُفَّ كما تلف العمامة.

http://www.almaany.com/quran/39/5/يُكَوِّرُ/



الليل و النهار ينتج عن دوران الأرض حول نفسها ولا علاقة له مطلقاً بشكل الأرض فقد تكون الارض على اي شكل مربعة او مخروطيّة هرميّة او مسطحة ..ويمر عليها الليل والنهار نتيجة دورانها !!

وهذ الحديث الثابت في الصحيحين يوضح بشكل لا يدع مجالاً للنقاش مقصد قوله في التكويّر

" الشمس والقمر ثوران مكوران في النار يوم القيامة "

و المقصود بالحديث دا انّ القمر والشمس سيمسخون يوم القمامة الى ثورين وسيذهب نورهما فكيف يكونان ثوران و كرتان بآن واحد ؟؟؟ والذي قاله الذي قاله لتحقير والاستخفاف بالهة القريشيين - الشمس و زوجها الله القمر مجرد هلوسة وتحقير لمعتقدات الآخرين من قبل مجرم مختل عقليا محب للقتل و سفك الدماء ونكاح النساء او اغتصابهن لا اكثر

و القول بأن للشمس مطلعاً على الأرض و مغرباً لها وأن للأرض مشارق و مغارب و أن لها أطرافاً ينقضها ذاك. لا يعني البتّة كروية الأرض بل يؤكّد على بسطها , حسب النظرة القديمة لشكل الأرض و الكون ..

https://en.wikipedia.org/wiki/Flat_Earth





الرد :

الأرض في القرآن الكريم مبسوطة ومسطحة بنظر الشخص الواقف عليها والناظر اليها ، فذلك شيء صحيح .

لننظر الى الآيات الكريمة من سورة الغاشية

أَفَلَا يَنظُرُونَ إِلَى الْإِبِلِ كَيْفَ خُلِقَتْ (17) وَإِلَى السَّمَاءِ كَيْفَ رُفِعَتْ (18) وَإِلَى الْجِبَالِ كَيْفَ نُصِبَتْ (19) وَإِلَى الْأَرْضِ كَيْفَ سُطِحَتْ (20) سورة الغاشية

واضح جدًا، ان الآيات الكريمة تخاطب الناس على وجه الأرض، وتدعوهم الى تأمل إبداع الخالق، ومن ذلك حُسن خِلقَة الإبل وكيف أن السماء مرفوعة وأن الجبال منصوبة ، وكيف أن الأرض مسطحةً.

ودائما تبدو الأرض مسطحة لمن يتأملها وهو واقف على سطحها !

بل إن الآية دائمًا صحيحة ، فقط في حالة الارض الكروية. فلو كانت الأرض مسطحه دائمًا، لما كانت الآية صحيحة لمن يقف قبل حافة الأرض بقليل وينظر أمامه ، لأنه سيراها مبتورة غير مسطحةً !

بالنسبة للآية الكريمةً "وَإِذَا الْأَرْضُ مُدَّتْ" فهي تتحدث عّن حال الأرض يوم القيامة وليس في حياتنا هذه ، أنظروا الآيات الكريمة من سورة الانشقاق ؛

إِذَا السَّمَاءُ انشَقَّتْ (1) وَأَذِنَتْ لِرَبِّهَا وَحُقَّتْ (2) وَإِذَا الْأَرْضُ مُدَّتْ (3) وَأَلْقَتْ مَا فِيهَا وَتَخَلَّتْ (4) وَأَذِنَتْ لِرَبِّهَا وَحُقَّتْ (5).

ومع ذلك مدَّ الأرض يوم القيامة ، هو شيء غيبي ، لا نعلم عنه شيئًا ، هل ستكون الارض مسطحة ام كروية ؟ الله أعْلَم . لكن لاحظ ، عزيزي القارئ ،كيف يتلاعب المستلحد باستعمال الآيات كي يبث سمومه وأكاذيبه !



أما الآية الكريمة من سورة الرعد

أَوَلَمْ يَرَوْا أَنَّا نَأْتِي الْأَرْضَ نَنقُصُهَا مِنْ أَطْرَافِهَا ۚ وَاللَّهُ يَحْكُمُ لَا مُعَقِّبَ لِحُكْمِهِ ۚ وَهُوَ سَرِيعُ الْحِسَابِ (الرعد 41)

فهي إشارة واضحة على كروية الأرض وليس سطحيتها ! فطرف الأرض او آخرها يبدو ناقصًا للمتأمل الواقف عليها ، فقط في حالة الكرة ! لأنه لا يوجد لسطح الكرة طرف او حدّ ، وسيتراءى دائمًا لمن يسعى للوصول الى آخر الارض ان طرفها يخدعه ويبتعد عنه ، وذلك ممكن فقط في حالة الشكل الكروي !



وأما الآية الكريمة من سورة نوح

(والله جعل لكم الأرض بساطا)

فقد وردت ايضًا في سياق مخاطبة رب العزة للبشر ، ودعْوَتهم لتأمل سطح الأرض ، فالآية معطوفة على ( الم تروا )

في بداية الآيات، واليكم السياق كاملًا

أَلَمْ تَرَوْا كَيْفَ خَلَقَ اللَّهُ سَبْعَ سَمَاوَاتٍ طِبَاقًا (15) وَجَعَلَ الْقَمَرَ فِيهِنَّ نُورًا وَجَعَلَ الشَّمْسَ سِرَاجًا (16) وَاللَّهُ أَنبَتَكُم مِّنَ الْأَرْضِ نَبَاتًا (17) ثُمَّ يُعِيدُكُمْ فِيهَا وَيُخْرِجُكُمْ إِخْرَاجًا (18) وَاللَّهُ جَعَلَ لَكُمُ الْأَرْضَ بِسَاطًا (19)



مرة اخرى، لاحظوا تلاعب الحاقد ومحاولته خداع القراء!

فيما يتعلق بالآيات الكريمة التالية

وَالْأَرْضَ بَعْدَ ذَٰلِكَ دَحَاهَا (30) أَخْرَجَ مِنْهَا مَاءَهَا وَمَرْعَاهَا (31) وَالْجِبَالَ أَرْسَاهَا (32) مَتَاعًا لَّكُمْ وَلِأَنْعَامِكُمْ (33) / سورة النازعات

وَالْأَرْضَ مَدَدْنَاهَا وَأَلْقَيْنَا فِيهَا رَوَاسِيَ وَأَنْبَتْنَا فِيهَا مِنْ كُلِّ شَيْءٍ مَوْزُونٍ (الحجر: 19 )



فهذه الآيات الكريمة عبارة عن إعجاز علمي، بدأت البشرية باكتشافه منذ أواسط القرن 19 وحتى أيامنا هذه .

فالآيات الكريمة، تتحدث عن أن الله عز وجل ، دحا : مدّ وبسط وجه الكرة الأرضية وتربط الآيات الكريمة عملية بسط السطح بخروج الماء والمواد الغذائية من جوفها ( أَخْرَجَ مِنْهَا مَاءَهَا وَمَرْعَاهَا ) وبنشوء الجبال وبروزها ( وَالْجِبَالَ أَرْسَاهَا ) ( وَالْأَرْضَ مَدَدْنَاهَا وَأَلْقَيْنَا فِيهَا رَوَاسِيَ ). وهي أمور أكدها العلم الحديث !

فآخر أبحاث الجيولوجيا والفضاء، تشير الى ان الماء على سطح الكرة الأرضية مصدره لُبّ الكرة الأرضية ، ويُعتقد ان نيازك وكويكبات حاملة لكميات ضخمة من الجليد ضربت الكرة الأرضية حين كان كوكب الأرض يتكوّن وينشأ ، بالإضافة إلى انجذاب جزيئات الهيدروجين المنتشرة في غبار (السديم الشمسي ) الى لُبّ الكرة الأرضية وتفاعلها مع جزيئات الأوكسجين لتنتج المزيد من المياه ,في باطن الأرض ( النواة core وطبقة الوشاح mantle ).

Hydrogen and noble gases from the solar nebula were drawn to the planetary embryo,
Hydrogen was then drawn
toward the center of the Earth – a process called isotopic fractionation. Hydrogen was delivered to the core through its attraction to iron, while much of the heavier isotope, deuterium, remained in the magma which eventually cooled to form the mantle



للمزيد إقرأوا هنا


اذًا فالماء على سطح الأرض، مصدره من باطن الأرض .

وبالنسبة للجبال:

فمن الثابت علميًا ان سطح الكرة الارضية مبني من ألواح ضخمة او صفائح tectonic plates ,تحمل فوقها اليابسة والمحيطات ، وأن الجبال تنشأ بسبب تصادم صفائح القشرة الارضية مع بعضها البعض .

للمزيد أقرأوا هنا


كذلك ، من المعروف علميًا انه على الرغم من الأهوال التي تتسبب بها البراكين للبشر ، ألا ان لها حسنات بيئية هامة ؛ فهي تزود سطح الكرة الأرضية بالماء والمواد الغذائية الضرورية لعالم النبات مثل الاوكسيجين ، السيليكا ، ثاني أكسيد الكربون ، ثاني أكسيد الكبريت ، الماچما وغيره



Volcanic eruptions result in ash being dispersed over wide areas around the eruption site. And depending on the chemistry of the magma from which it erupted, this ash will be contain varying amounts of soil nutrients. While the most abundant elements in magma are silica and oxygen, eruptions also result in the release of water, carbon dioxide (CO²), sulfur dioxide (SO²), hydrogen sulfide (H²S), and hydrogen chloride (HCl), amongst others.



وللمزيد إقرأوا هنا


يتضح ،بناءً على ما سبق ، ان الآيات التي يحاول المستلحد استعمالها للطعن في القرآن الكريم . في الواقع تطعن كذبه وافتراءه وتثبت سماوية الذكر الحكيم وانه من عند عزيز مقتدر !

فالإعجاز العلمي فيها واضح تماماً ؛

الله ، دحا سطح الأرض؛ جعله مكوّن من الواح ممتدة متحركة متصادمة ، وبسبب التصادم خرجت المياه والمواد الغذائية الضرورية لعالم الأحياء، وكذلك نشأت الجبال وبرزت .



وَالْأَرْضَ بَعْدَ ذَٰلِكَ دَحَاهَا (30) أَخْرَجَ مِنْهَا مَاءَهَا وَمَرْعَاهَا (31) وَالْجِبَالَ أَرْسَاهَا (32) (33) / سورة النازعات

بصدد الادعاء حول تكوير الليل والنهار

فالآية الكريمة تقول:

"خَلَقَ السَّمَاوَاتِ وَالْأَرْضَ بِالْحَقِّ ۖ يُكَوِّرُ اللَّيْلَ عَلَى النَّهَارِ وَيُكَوِّرُ النَّهَارَ عَلَى اللَّيْلِ ۖ وَسَخَّرَ الشَّمْسَ وَالْقَمَرَ ۖ كُلٌّ يَجْرِي لِأَجَلٍ مُّسَمًّى ۗ أَلَا هُوَ الْعَزِيزُ الْغَفَّارُ "

(الزمر 5)

استعلمت الآية الكريمة، كلمة ( يكوّر )

والحقيقة ، اننا لا نحتاج لكثير من المعرفة والعلم باللغة العربية حتى نفهم ان يكوّر مشتقة من نفس المصدر اللغوي لكلمة ( كرة )!

وبالرغم من ذلك ، اليكم معنى كلمة ( يكوّر ) من معجم اللغة العربية

كَوَّرَ العِمَامَةَ عَلَى رَأْسِهِ : لَفَّهَا ، أَدَارَهَا

كَوَّرَ الشيءَ : لفَّه على جهة الاستدارة

كَوَّرَ المتاعَ : أَلقى بعضَه على بعضٍ أَو جَمَعَه


فالآية الكريمة تصف شكل تعاقب الليل والنهار على بعضهما البعض بالتكوير ، اي ان حلول الظلام يأخذ شكلًا مكوّرًا وكذلك حلول النهار يتخذ أيضا شكلًا مكوّرًا - لأن كل واحد منهما يُغَطي تقريباً نصف الكرة الأرضية كل يوم ، بل كل لحظة ولحظة.

مرة أخرى ، ومثل كل مرة ، يقصد الحاقد على الإسلام مهاجمة القرآن الكريم والطعن فيه ،لكنه دون ان يقصد ، يقوم بتنبيه الناس الاعجاز للعلمي المكنون في الذكر الحكيم فيؤكد سماوية القرآن وكونه منزلًا من لدن حكيمٍ خبيرٍ!

بالنسبة للحديث المذكور

"الشمس والقمر ثوران مكوران في النار يوم القيامة " فهو حديث ضعيف . وليس كما يكذب ، المفتري الحاقد ، فالحديث غير مذكور في الصحيحين !

والحديث في صحيح البخاري، مذكور بهذا النص " الشمس والقمر مكورِّان يوم القيامة"


والحديث الشريف المذكور في صحيح البخاري ، مطابق للآية الكريمة في سورة التكوير "إِذَا الشَّمْسُ كُوِّرَتْ (1)" والمعنى واضح ، وهو أنه يوم القيامة سيكون كل من ضوء الشمس ونور القمر؛ ضوءًا او نورًا مكورًا : يعني ملفوف بشكل كروي ، الشمس لن تضيء في شتى الاتجاهات وكذلك القمر لن ينير في كل اتجاه، بل سيبقى ضوء /نور الشمس والقمر ملازم لهما ومحيط بهما - وفِي ذلك إشارة الى كل من عبد الشمس او القمر الى انهما مجرد مخلوقين لا غير !

بالنسبة لمشارق الأرض ومغاربها، فقد ورد في القرآن الكريم

وَأَوْرَثْنَا الْقَوْمَ الَّذِينَ كَانُوا يُسْتَضْعَفُونَ مَشَارِقَ الْأَرْضِ وَمَغَارِبَهَا الَّتِي بَارَكْنَا فِيهَا (الأعراف 137)

وفِي الحديث الشريف

"إن الله قد زوى لي الأرض فرأيت مشارقها ومغاربها وإن أمتي سيبلغ ملكها مَا زوى لي منها" رواه مسلم

وهو شيء صحيح، فالشمس لا تشرق ولا تغرب من نقطةٍ واحدةٍ، فكلّ يوم تشرق من نقطةٍ تختلف عن سابقتها وتغرب بنفس النمط. ولذلك تتغير مدة النهار ومدة الليل وتتغير أوقات الصلاة.

وسبب ذلك هو أن محور دوران الأرض حول نفسها لمسار دوران الكرة الأرضية حول الشمس، هو محور مائل وليس عموديًا . ويميل محور دوران الارض بزاوية 23.44 درجة .

ولذلك هناك فترات يكون فيها نهار تام دون ليل في القطب الشمالي ويقابله ظلام دامس دون نهار في القطب الجنوبي !

وللمزيد اقرأوا هنا


كما ترى ، عزيزي القارئ ، ادعاءات الطاعنين في القرآن الكريم ، عبارة عن تخبيصات وكذب ، لا غير ، ولا شك أنك لاحظت مستواهم الهابط في التطاول على الذات الإلهية وشخص الرسول الكريم ، ويومًا ما سنقف امام الواحد الدّيان

(وَسَيَعْلَمُ ٱلَّذِينَ ظَلَمُوٓاْ أَىَّ مُنقَلَبٍۢ يَنقَلِبُونَ)

الافتراء:

9 - خلق العظام قبل اللحم يالقرآن :

" فَخَلَقْنَا الْمُضْغَةَ عِظَامًا فَكَسَوْنَا الْعِظَامَ لَحْمًا " المؤمنون :14

الآحاديث الصحيحة :.

عن عبد اللّه بن مسعود قال: حدثنا رسول اللّه :

" إن أحدكم يجمع خلقه في بطن أمه أربعين يومًا نطفة، ثم يكون علقة مثل ذلك، ثم يكون مضغة مثل ذلك، ثم يرسل إليه الملك فينفخ فيه الروح، ويؤمر بأربع كلمات: بكتب رزقه، وأجله، وعمله، وشقي أو سعيد "

عن حذيفة بن أسيد قال :...

"إذا مر بالنطفة ثنتان وأربعون ليلة، بعث اللّه إليها ملكًا، فصورها، وخلق سمعها وبصرها، وجلدها ولحمها وعظامها. ثم يقول: يا رب، أذكر أم أنثى ؟ فيقضي ربك ما شاء،

ما يقوله العلم الحديث و الطب المثبت ..

النطفة : تعيش 72 ساعة بجسم المرأة كحد أقصى

جنس الجنين (ذكر أو انثى) يتحدد في لحظة التلقيح عند تكوّن (الزيجوت) واكتمال الـ (DNA) , و يحدد ذلك وجود الكروموسوم .

في اليوم التاسع يبدأ (اللحم ) بالتشكّل ذلك عندما تلتحم البيضة الملقّحة بجدار الرحم وتبدأ بامتصاص الغذاء

في اليوم 21 : يظهر الجنين و يمكن تسجيل النبضات القلبية عندها و يبدأ الدوران الدموي

السمع و البصر و العظام (العينين و الأذنين و العمود الفقري [اول العظام التي تبدأ بالظهور تدريجيّاً]) تبدأ بالتشكّل في اليوم الـ 28

عظم الحنك يبدأ بالتكوّن ابتداءاً من اليوم الـ 50

في الاسبوع العاشر يبدأ تشكل غضاريف الجنين و عظامه الباقيّة...

راجع :

http://www.pregnancycenterwest.org/fetal-development.html

http://www.hisbranches.org/eo/fetal/timeline.htm

http://webspace.ship.edu/cgboer/genpsyfetaldev.html

http://lambtonrighttolife.org/pro-life-issues/fetal-development/





الرد :



يُصَّور المفتري الأمور وكأن هناك تناقضًا بين آيات القرآن الكريم وعلم الأجنة. والحقيقة أن الحقائق العلمية تتطابق مع آيات الذكر الحكيم بشكل مذهل، فالعظام تتمايز قبل خلايا العضلات، مثلما تذكر الآية الكريمة " فَخَلَقْنَا الْمُضْغَةَ عِظَامًا فَكَسَوْنَا الْعِظَامَ لَحْمًا " المؤمنون:14

فحين نطالع علم الأجنة نرى أن الجنين

في الأسبوع الثالث بعد الإخصاب ( او الأسبوع الخامس للحمل والذي يُحسب ابتداء من تاريخ آخر دورة شهرية ) ، يكون عبارة عن حويصلة مجوفه ذات ثلاث طبقات من الأغشيه Germ layers مرتبه فوق بعضها البعض. تسمى طبقة الخلايا الأكثر قرباً لجدار الرحم ب(الإكتودرم) ectoderm والطبقة الوسطى ب(الميزودرم) mesoderm وتسمى الطبقة الأكثر قرباً لفراغ التجويف ب(الإندورم) endoderm. تتمايز من الطبقة الوسطى ( الميزودرم ) خلايا مسؤوله عن بناء الجسم تسمى ب(الجسدية) somites والتي بدورها تنقسم الى ثلاث أنواع من الخلايا الجذعية المتخصصة : dermatome تتمايز إلى الجلد والدهن. myotome تتمايز إلى خلايا عضليه ( عضلات الهيكل العظمي ، القلب ، الأوعية الدموية وغيرها ) . Sclerotome تتمايز إلى خلايا العظام، الغضاريف والأوتار ( وهي الألياف التي تربط بين العضلات أو بين العضلات والعظام ). أول ما يتمايز هي خلايا ال sclerotome وبذلك تتخلق خلايا العظام قبل خلايا العضلات ( اللحم ) .



المصادر





بالنسبة للحديث الشريف حول نمو الجنين ، فالحديث لا يقول ان النطفة تدوم على حالها في رحم المرأة أربعين ليلةً ! بل الحديث يعني أن النطفة تتخلق وتمر بمرحلة العلقة والمضغة وتَخَّلُق العظام - كل هذا المراحل تتم خلال أربعين ليلة من الحمل .

وطبعًا من أجل ان يتلاعب الحاقد على الإسلام بفهم الحديث ويُمَوه القارئ فانه لا يذكر الحديث من صحيح مسلم والذي يوضح ما ورد في حديث ابن مسعود ، اليكم الحديث من صحيح مسلم؛



" إنَّ أَحَدَكُمْ يُجْمَعُ خَلْقُهُ في بَطْنِ أُمِّهِ أَرْبَعِينَ يَوْمًا، ثُمَّ يَكونُ في ذلكَ عَلَقَةً مِثْلَ ذلكَ، ثُمَّ يَكونُ في ذلكَ مُضْغَةً مِثْلَ ذلكَ، ثُمَّ يُرْسَلُ المَلَكُ فَيَنْفُخُ فيه الرُّوحَ، وَيُؤْمَرُ بِأَرْبَعِ كَلِمَاتٍ: بِكَتْبِ رِزْقِهِ، وَأَجَلِهِ، وَعَمَلِهِ، وَشَقِيٌّ، أَوْ سَعِيدٌ،...

الحديث أعلاه واضح ، ويذكر أن المراحل التالية ، تتم في غضون أربعين يومًا وتوضح هذا الأمر، العبارة المتكررة في الحديث ( في ذلك ) والمراحل هي : جمع الخلق في بطن الام : أي التقاء الحيوان المنوي والبويضة لإنتاج البويضة المخصبة، ثم العلقة ، وتليها مرحلة المضغة .

ولو قرأنا بعدها الحديث الصحيح لابن مسعود والذي استشهد به المفتري ، لسهل علينا رؤية التطابق بين الحديثين

"إنَّ أحدَكم يُجْمَعُ خَلْقُهُ في بطنِ أمِّه أربعينَ يومًا نطفةً ، ثم يكونُ علقةً مثلَ ذلك ، ثم يكونُ مضغةً مثلَ ذلك ، ثم يبعثُ اللهُ إليه ملَكا ، ويُؤمرُ بأربعِ كلماتٍ ، ويُقالُ له : اكتبْ عملَه ، ورزقَه ، وأجلَه ، وشقيٌّ أو سعيدٌ"

ويؤكد الحديث الثاني ( أدناه )، الذي ذكره المدلس، يؤكد صحة الحديث السابق لان الحديث يذكر تخلق العظم والجلد واللحم خلال ٤٢ ليلةً "إذَا مَرَّ بالنُّطْفَةِ ثِنْتَانِ وَأَرْبَعُونَ لَيْلَةً، بَعَثَ اللَّهُ إلَيْهَا مَلَكًا، فَصَوَّرَهَا وَخَلَقَ سَمْعَهَا وَبَصَرَهَا وَجِلْدَهَا وَلَحْمَهَا وَعِظَامَهَا، ثُمَّ قالَ: يا رَبِّ أَذَكَرٌ أَمْ أُنْثَى؟ فَيَقْضِي رَبُّكَ ما شَاءَ" رواه مسلم . فواضح اذًا ان الحديث الشريف قَصَد حدوث كل مراحل تخلق الجنين خلال أربعين يومًا وليس حدوث كل مرحلة على حدة في أربعين يومًا !



وبما يتعلق بجنس المولود

فالحديث الشريف لا يعني ولا يمكن ان يعني تحديد جنس المولود بحسب نوعية الكروموزومات Y, X. فالحمض النووي لم يكن معروفاً آنذاك ! وانما قصد تحديد جنس المولود بحسب الاعضاء التناسلية، اما ذكرية واما أنثوية . وهذا بحد ذاته اعجاز مذهل فالعلم الحديث يقول صراحةً انه يمكن رؤية الاعضاء الجنسية عند الجنين فقط بعد مرور 7-6 أسابيع من فترة الحمل ! بالضبط مثلما ذكر الحديث الشريف "إذَا مَرَّ بالنُّطْفَةِ ثِنْتَانِ وَأَرْبَعُونَ لَيْلَةً.."

واليكم ترجمة فقرة من مقال علمي ؛

" في بداية تخلق ونمو الغدد التناسلية عند الجنين ، يبقى الجنين غير متمايزًا ، أي ان كل الأجنة تكون تتشابه أعضائها الجنسية ، كلها تبدو أنثوية، وبعد 6-7 أسابيع من الحمل ، تتم عملية ترجمة الجينات في كرموزوم Y فتحدث عمليات تؤدي الى ظهور الخصيتين



During early development the gonads of the fetus remain undifferentiated; that is, all fetal genitalia are the same and are phenotypically female. After approximately 6 to 7 weeks of gestation, however, the expression of a gene on the Y chromosome induces changes that result in the development of
the teste



واليكم الرابط




بالنسبة لمراحل نمو الجنين وتخلق أعضاءه ، فالمستلحد يستمر بنثر المعلومات بطريقة تبدو وكأنها تتناقض مع آيات القرآن الكريم ، والحقيقة هي العكس فعلم الأجنة يؤكد ما ذكرته آيات الذكر الحكيم ، واليكم أمثلة :

تقول آيات القرآن ان حاسة السمع تنشأ قبل حاسة البصر وكذلك أول عضو يكتمل تخلقه في الجنين هو القلب. إقرأ معي ، عزيزي القارئ ،الآيات التالية وتبين بنفسك

وَهُوَ الَّذِي أَنشَأَ لَكُمُ السَّمْعَ وَالْأَبْصَارَ وَالْأَفْئِدَةَ ۚ ( المؤمنون 78)

وَجَعَلْنَا لَهُمْ سَمْعًا وَأَبْصَارًا وَأَفْئِدَةً ( الأحقاف 26)

نفس الأمر تقوله الحقائق العلمية، ففي الأسبوع الثالث بعد الإخصاب ، تترتب بعضها فوق بعض داخل النطفة ، ثلاث أغشيه من الخلايا الجذعية . تسمى الطبقة الاكثر قرباً لبطانة الرحم بالإكتودرم ectoderm

وتتمايز منها من منطقة الدماغ لويحات عصبية، أولها اللويحة السمعية otic placode والتي ستتطور الى أعصاب جهاز السمع وأعضاؤه ، وفِي نهاية الأسبوع الثالث وبداية الأسبوع الرابع تظهر اللويحة البصرية optic placode ومنها ستتطور أعصاب الرؤية والعينين . في اليوم التاسع عشر ينمو من طبقة الأغشية الوسطى الميزودرم mesoderm أنبوبان دمويان endocardial tubes ينموان الواحد بمحاذاة الآخر ثم يمتزجان ويكونان وعاءً دموياً سيتطور الى قلب ، وفِي الأسبوع الرابع في حدود اليوم الثاني والثالث والعشرين يبدأ القلب بالخفقان وضخّ الدم .

والقلب هو أول عضو تكتمل بنيته ويبدأ بالعمل في جسم الجنين.

وبذلك تتخلق حاسة السمع قبل حاسة البصر ويتبع ذلك مباشرة اكتمال نمو القلب وبداية عمله

المصادر





ولنكمل مع المزيد من الإعجازات العلمية المذهلة ؛

نحن نعلم أن بويضة الأم لا تحدد جنس الجنين لان نوع كروموزوم الجنس في البويضة دائمًا يكون X. لذلك جنس الجنين يتحدد فقط بحسب نوع الكروموزوم الموجود في الحيوان المنوي . فان كان من نوع Y فالجنين يكون ذكرًا وإن كان كروموزوم X فجنس الجنين يكون أنثى .

الآن تأملوا الآية الكريمة التالية ، ولاحظوا كيف نسبت جنس المولود ( ذكر أو أنثى ) الى المني ( والذي مصدره فقط الرجل ) !

"وَأَنَّهُ خَلَقَ الزَّوْجَيْنِ الذَّكَرَ وَالْأُنثَىٰ (45) مِن نُّطْفَةٍ إِذَا تُمْنَىٰ (46)" سورة النجم



واليكم مثالًا آخر ؛ ذكرت ايات القرآن الكريم ان الجنين في طور تخلقه يمر بمرحلة مضغة مُخَّلَقة وغير مُخَّلقة

"يَا أَيُّهَا النَّاسُ إِن كُنتُمْ فِي رَيْبٍ مِّنَ الْبَعْثِ فَإِنَّا خَلَقْنَاكُم مِّن تُرَابٍ ثُمَّ مِن نُّطْفَةٍ ثُمَّ مِنْ عَلَقَةٍ ثُمَّ مِن مُّضْغَةٍ مُّخَلَّقَةٍ وَغَيْرِ مُخَلَّقَةٍ لِّنُبَيِّنَ لَكُمْ ۚ وَنُقِرُّ فِي الْأَرْحَامِ مَا نَشَاءُ إِلَىٰ أَجَلٍ مُّسَمًّى " الحَجّ (5)

تخبرنا حقائق علم الأجنة أنه بعد مرور 3-4 ايام من الإخصاب ، يصل حجم البويضة المخصبة ( الزيچوتا ) الى 16 خليه وتترتب بشكل عنقود صغير شبيه بثمرة توت الشجر لذلك يطلق عليها اسم ( التوتية ) morula . تتوالى الانقسامات ما بين اليوم الرابع والخامس ليصل العدد الى 50-100 خلية، حينئذ تكون مجموعة الخلايا ( النطفة ) قد وصلت الى تجويف الرحم ،يتكون من داخل النطفة تجويف يسمى ( الكيسة الأرمية ) blastocyst تترتب في داخله وفي الجهة القريبة لبطانة الرحم حوالي 30 خليه جذعية ذات قدرة على التمايز والتخلق تسمى( كتلة الخلايا الداخلية ) ICM inner cell mass قسم منها سوف يتطور الى الأغشية الجنينية Germ layers والقسم الباقي سيبقى رصيداً مدخراً طوال العمر لترميم وتعويض ما يتلف من خلايا الأعضاء Adult stem cells .

وبالإمكان سرد المزيد والمزيد من التطابق المذهل بين آيات القرآن الكريم وبين علم الأجنة

ويبقى الحق حقًا ويبقى الباطل باطلًا

الافتراء:

10 - تصوّر الشهب الخاطئ في القرآن

- الشهب في القرآن هي كواكب و نجوم !!

{ وَلَقَدْ زَيَّنَّا السَّمَاء الدُّنْيَا بِمَصَابِيحَ وَجَعَلْنَاهَا رُجُومًا لِّلشَّيَاطِينِ } الملك : 5

{ إِنَّا زَيَّنَّا السَّمَاء الدُّنْيَا بِزِينَةٍ الْكَوَاكِبِ , وَحِفْظًا مِّن كُلِّ شَيْطَانٍ مَّارِدٍ , لَا يَسَّمَّعُونَ إِلَى الْمَلَإِ الْأَعْلَى وَيُقْذَفُونَ مِن كُلِّ جَانِبٍ , دُحُورًا وَلَهُمْ عَذَابٌ وَاصِبٌ , إِلَّا مَنْ خَطِفَ الْخَطْفَةَ فَأَتْبَعَهُ شِهَابٌ ثَاقِبٌ } . الصافات:6-10

{ وَأَنَّا لَمَسْنَا السَّمَاء فَوَجَدْنَاهَا مُلِئَتْ حَرَسًا شَدِيدًا وَشُهُبًا , وَأَنَّا كُنَّا نَقْعُدُ مِنْهَا مَقَاعِدَ لِلسَّمْعِ فَمَن يَسْتَمِعِ الْآنَ يَجِدْ لَهُ شِهَابًا رَّصَدًا } . الجن:8-9

{ وَلَقَدْ جَعَلْنَا فِي السَّمَاء بُرُوجًا وَزَيَّنَّاهَا لِلنَّاظِرِينَ , وَحَفِظْنَاهَا مِن كُلِّ شَيْطَانٍ رَّجِيمٍ , إِلاَّ مَنِ اسْتَرَقَ السَّمْعَ فَأَتْبَعَهُ شِهَابٌ مُّبِينٌ } الحجر :16-18 .......

{ وَالنَّجْمِ إِذَا هَوَى } / ( النجم : 1) ...

التفاسير :

http://www.grenc.com/show_article_main.cfm?id=21517

الشهب علميّاً :

النَيزَك جسيم يوجد في النظام الشمسي يتكون من حطام الصخور وقد يكون في حجم حبيبات الرمل الصغيرة أو في حجم صخرة كبيرة. ( المسار المرئي للنيزك الذي يدخل الغلاف الجوي الخاص بكوكب الأرض يعرف باسم الشهاب ) ، أما إذا وصل إلى سطح الأرض، فإنه في هذه الحالة يعرف باسم الحجر النيزكي ., هي لا تنطلق من ما يسمّى السماء السابعة " كما بقوله الْمَلَإِ الْأَعْلَى " انما تدور حول الشمس بمدارات مختلفة . و تدخل الأرض عند اختراقها مجال جاذبيتها لتشتعل ضمن الغلاف الجوي للأرض " الأيونوسفير" ولا تنشأ من النجوم (زينة السماء) كما يقول علماء وشيوخ المسلمين بل ان النجوم مجرّد كتل هائلة من الغازات المتوهجة المشتعلة ليست صلبة و لاعلاقة لها بالشهب !!. وقد أشار القرآن صراحةً الى أن النجوم أو المصابيح !!! كما وصفها القرآن هي ذاتها الشهب كما في الآيات السابقة هو أمر لا يمكن تكذيبه كما يقوم بعض البلهاء بالتحايل على ذلك فالنجوم هي ذاتها زينة السماء التي تضيئ كالمصابيح كما ذكر ذلك لقرآن صراحة وجميع مفسّريّه وإن لم تكن كذلك فما عساها أن تكون مصابيح الزينة التي تملأ السماء و يراها الناظرين إذاً !!؟؟

وتبعاً لذلك بما أنَّ السماء الدنيا " الأيونوسفير " هي ذاتها من تهبط بها المصابيح التي نراها على هيئة الشهب لا يمكن القول بأن كلمة " مُوسِعُونَ " في آية وَالسَّمَاءَ بَنَيْنَاهَا بِأَيْدٍ وَإِنَّا لَمُوسِعُونَ / الذاريات : 47 تعني التوسع كما يدّعي مدّعي الاعجاز العلمي بالقرآن معتمدين على احدى النظريات للكون ومهمشي جميع الاثباتات العلمية التي تنقض قرآنهم

فمعناها الحقيقي هو قادرون أو ذو سعة لأنَّها وردت في القرآن من دون التشديد على حرف السين بها ليقوموا بتحريف قرآنهم على حساب اثبات تطابقه مع نظريّة علميّة لا تدعمه !





الرد

كالعادة ، الكلام اعلاه عبارة عّن خلط ولخبطة ومحاولة للتدليس على القرآن الكريم

١- لم تقل آيات القرآن الكريم أن النيازك والشهب تنشأ من النجوم ، وأيضًا لم تقل أن النجوم هي ذاتها الشهب والنيازك ! هذا افتراء من عند الحاقد على الاسلام. لنقرأ الآيات معًا

"وَلَقَدْ زَيَّنَّا السَّمَاءَ الدُّنْيَا بِمَصَابِيحَ وَجَعَلْنَاهَا رُجُومًا لِّلشَّيَاطِينِ ۖ وَأَعْتَدْنَا لَهُمْ عَذَابَ السَّعِيرِ "(الملك 5).

لفظة (جعلناها) تعود على ( السَّمَاءَ الدُّنْيَا ) وليس على ( المَصَابِيحَ)!

وسيتضح المعنى اكثر حين نقرأ الآيات من سورة الصافات 

إِنَّا زَيَّنَّا السَّمَاءَ الدُّنْيَا بِزِينَةٍ الْكَوَاكِبِ (6) وَحِفْظًا مِّن كُلِّ شَيْطَانٍ مَّارِدٍ (7) لَّا يَسَّمَّعُونَ إِلَى الْمَلَإِ الْأَعْلَىٰ وَيُقْذَفُونَ مِن كُلِّ جَانِبٍ (8) دُحُورًا ۖ وَلَهُمْ عَذَابٌ وَاصِبٌ (9) إِلَّا مَنْ خَطِفَ الْخَطْفَةَ فَأَتْبَعَهُ شِهَابٌ ثَاقِبٌ (10)

فالآيات واضحة وتُبَيَّن أن الشيء الذي يُرْجَم به هو الشُهُب ، وليس الكواكب او النجوم ( المصابيح )!

" إِلَّا مَنْ خَطِفَ الْخَطْفَةَ فَأَتْبَعَهُ شِهَابٌ ثَاقِبٌ "



٢- ويستمر تخبيص المفتري وتدليسه حين يَزْعُم ان "الايونوسفير " ionosphere هي السماء الدنيا ! والحقيقة هي ان "الايونوسفير" هي طبقة من طبقات الغلاف الجوي المحيط بكوكب الارض، وهي الطبقة المتأينة بسبب أشعة الشمس وترتفع عن سطح الأرض 75-1000 كيلومترًا ،

وللمزيد اقرأوا هنا


اما السماء الدنيا ، فهي كل الكون المنظور وغير المنظور بما فيه من بلايين المجرات ومن ضمنها كوكبنا ، وقد اخبرنا رب العزة في قرآنه الكريم ان السماء الدنيا هي السماء الاولى وأنها مُغَلَّفة بست سموات فوقها ، اي أن عدد السموات هو سبع

"هو الَّذِي خَلَقَ سَبْعَ سَمَاوَاتٍ طِبَاقًا ۖ مَّا تَرَىٰ فِي خَلْقِ الرَّحْمَٰنِ مِن تَفَاوُتٍ ۖ فَارْجِعِ الْبَصَرَ هَلْ تَرَىٰ مِن فُطُورٍ " (المُلْك 3)

والعلم لا يستطيع الإحاطة بالسماء الدنيا - الاولى ولا رصدها كلها ، لانه عاجز وقاصر عن ذلك ، فالكون عمره ١٣.٨ مليار سنة وهو في تمدد وتوسع والبشرية لا تستطيع الا رصد ما يصلها من ضوء . وقد يكون حجم الكون المرصود قريب جدًا من الصفر !

وعلى أية حال فطبقة الايونوسفير ليست هي السماء الدنيا كما يهذي المستلحد .





وكما هو الحال في كل مرة، يأتي الحاقد ليفتري على القرآن. فيلفت نظرنا الى الاعجاز العلمي المكنون في آيات القرآن الكريم . ذلك أن آيات القرآن أشارت الى وجود علاقة بين قَذْفْ الشهب والنيازك وبين الكواكب والنجوم. لنطالع معًا مرةً ثانية الآيات الكريمة

"وَلَقَدْ زَيَّنَّا السَّمَاءَ الدُّنْيَا بِمَصَابِيحَ وَجَعَلْنَاهَا رُجُومًا لِّلشَّيَاطِينِ ۖ وَأَعْتَدْنَا لَهُمْ عَذَابَ السَّعِيرِ "(الملك 5)





" إِنَّا زَيَّنَّا السَّمَاءَ الدُّنْيَا بِزِينَةٍ الْكَوَاكِبِ (6) وَحِفْظًا مِّن كُلِّ شَيْطَانٍ مَّارِدٍ (7) لَّا يَسَّمَّعُونَ إِلَى الْمَلَإِ الْأَعْلَىٰ وَيُقْذَفُونَ مِن كُلِّ جَانِبٍ (8) دُحُورًا ۖ وَلَهُمْ عَذَابٌ وَاصِبٌ (9) إِلَّا مَنْ خَطِفَ الْخَطْفَةَ فَأَتْبَعَهُ شِهَابٌ ثَاقِبٌ (10)" سورة النجم

وقذف الكواكب والنجوم بالشهب هي حقيقة أكدتها علوم الفضاء ، فالنيازك والشهب ، تتأثر بجاذبية الكواكب والنجوم . وتتسبب الجاذبية بحرف مسار الشهب وقذفها الى مسارات اخرى ! واليكم اقتباس من موقع ناسا مع الترجمة والرابط .

الجاذبية الضخمة لكوكب " المشتري "

والاقتراب من كوكب "المريخ " او أي جسم آخر تُغَيِّر مسار الشهاب asteroid ، وذلك يتسبب بإبعاد ( الشهب ) من الحزام الرئيسي ( منطقة بين المشتري والمريخ تتواجد بها الشهب ) وقَذْفِها - hurling them في الفضاء في عدة اتجاهات !

Jupiter's massive gravity and occasional close encounters with Mars or another object change the asteroids' orbits, knocking them out of the main belt and hurling them into space in all directions

واليكم رابط المقال


ملاحظة: النيزك meteor هو كل شهاب asteroid قريب من الكرة الأرضية ويدخل مجالها


٣- بالنسبة لمعنى الآية "وَالسَّمَاءَ بَنَيْنَاهَا بِأَيْدٍ وَإِنَّا لَمُوسِعُونَ "( الذاريات 47)



فهي ايضًا اعجاز علمي مذهل، فقد أخبرنا الله في كتابه عما يدل على اتساع السماء، فلفظة -لَمُوسِعُونَ : اسم فاعل بصيغة الجمع لفعل أوسع، وهو يفيد الاستمرار . من جهة ثانية، في نهاية العشرينيات من القرن العشرين، استنتج عالم الفضاء " ادوين هابل " أن الكون موجود في حالة تمدد واتساع ، وأن المجرات تتباعد عن بعضها بسرعة متزايدة . وللمزيد اقرأوا هنا




سبحان الله ، يحاول الحاقدون الافتراء على القرآن الكريم ، فيثيرون انتباهنا الى جود اعجازات علمية مكنونة فيه ، فيزيد يقيننا ان ذلك الكتاب لاريب فيه هدى من رب العالمين





الافتراء:

11 - حركة الشمس حول الأرض بالقرآن

- “وَالشَّمْسُ تَجْرِي لِمُسْتَقَرٍّ لَهَا / يس :38

- فَإِنَّ اللَّهَ يَأْتِي بِالشَّمْسِ مِنَ الْمَشْرِقِ فَأْتِ بِهَا مِنَ الْمَغْرِبِ / البقرة 258

- رَبُّ الْمَشْرِقَيْنِ وَرَبُّ الْمَغْرِبَيْنِ / الرحمن 17

- وَتَرَى الشَّمْسَ إِذَا طَلَعَتْ تَزَاوَرُ عَنْ كَهْفِهِمْ ذَاتَ الْيَمِينِ وَإِذَا غَرَبَتْ تَقْرِضُهُمْ ذَاتَ الشِّمَالِ وَهُمْ فِي فَجْوَةٍ مِنْهُ / الكهف 17

- وَسَخَّرَ الشَّمْسَ وَالْقَمَرَ كُلٌّ يَجْرِي لِأَجَلٍ مُسَمّىً يُدَبِّرُ / الرعد 2

- والشمس والقمر كل في فلك يسبحون / الأنبياء: 33 يسبحون بين كلِّ الأجرام في الفضاء !!!

- حَتَّىٰ-;---;-- إِذَا بَلَغَ مَطْلِعَ الشَّمْسِ وَجَدَهَا تَطْلُعُ عَلَىٰ-;---;-- قَوْمٍ لَّمْ نَجْعَل لَّهُم مِّن دُونِهَا سِتْرًا / الكهف :(90)

- حَتَّى إِذَا بَلَغَ مَغْرِبَ الشَّمْسِ وَجَدَهَا تَغْرُبُ فِي عَيْنٍ حَمِئَةٍ / الكهف:86

- الشَّمْسُ وَالْقَمَرُ بِحُسْبَانٍ ( يجريان ) / الرحمن : 5

- «وسخر لكم الشمس والقمر دائبين» جاريين في فلكهما لا يفتران «وسخر لكم الليل» لتسكنوا فيه «والنهار» لتبتغوا فيه من فضله.

حركة الشمس هي التي تحدد الظل !! :

أَلَمْ تَرَ إِلَى رَبِّكَ كَيْفَ مَدَّ الظِّلَّ وَلَوْ شَاءَ لَجَعَلَهُ سَاكِنًا ثُمَّ جَعَلْنَا الشَّمْسَ عَلَيْهِ دَلِيلًا / الفرقان : 45

و هذا اعتقاد خاطئ فدوران الأرض حول نفسها هو من يحدد حركة الظل ولا علاقة لحركة الشمس حول المجرّة البطيئة جداً به !!!

- بعض الأحاديث الصحيحة التي تدل على جريان الشمس حول الأرض

عن أبي ذر قال “كنت مع النبي في المسجد عند غروب الشمس فقال “يا أبا ذر أتدري أين تغرب الشمس؟” قلت: “الله ورسوله أعلم”. قال “فإنها تذهب حتى تسجد تحت العرش، فذلك قوله: “وَالشَّمْسُ تَجْرِي لِمُسْتَقَرٍّ لَهَا“ قال “مستقرها تحت العرش”. !!

عن صفوان ابن عسال “إن من قِبَلِ مغرب الشمس بابًا مفتوحًا عرضه سبعون سنة، فلا يزال ذلك الباب مفتوحًا للتوبة حتى تطلع الشمس من نحوه. فإذا طلعت مِنْ نَحْوِهِ لَمْ يَنْفَعْ نَفْسًا إِيمَانُهَا لَمْ تَكُنْ آمَنَتْ مِنْ قَبْلُ أو كسبت في إيمانها خيرًا”.

وكما ورد في بيان الأصول لعلماء المسلمين الذي ذكرته سابقاً فقول القرآن بأن للأرض مشرق و مغرب يخالف القول بدوران الأرض و كرويّتها





الرد :

النص أعلاه عبارة عن تلاعب وتحريف لمعاني الآيات الكريمة والأحاديث الشريفة. فالقرآن الكريم لم يقل ان الشمس تدور حول الأرض وكذلك الاحاديث الشريفة لم تذكر ذلك, واليكم الرد بالتفصيل :

وَالشَّمْسُ تَجْرِي لِمُسْتَقَرٍّ لَّهَا ۚ ذَٰلِكَ تَقْدِيرُ الْعَزِيزِ الْعَلِيمِ (يس 38)

سبحان الله يأتي المفتري ليدلس على آيات القرآن الكريم ، فيشير من حيث لا يقصد ،الى وجود اعجاز علمي فقد ذكرت الآية الكريمة أن الشمس تجري : تسبح في الفضاء ، وهي حقيقة علمية تجلت لنا فقط في عصرنا هذا ، فنحن نعلم أن الشمس تدور حول مركز المجرة " درب التبانة "

وتستغرق الشمس 226 مليون سنة، كي تتم دورتها ! وتجري الشمس في الفضاء بسرعة 783 كيلومترًا في الساعة ! وللمزيد من المعلومات اليكم هذا الرابط


"أَلَمْ تَرَ إِلَى الَّذِي حَاجَّ إِبْرَاهِيمَ فِي رَبِّهِ أَنْ آتَاهُ اللَّهُ الْمُلْكَ إِذْ قَالَ إِبْرَاهِيمُ رَبِّيَ الَّذِي يُحْيِي وَيُمِيتُ قَالَ أَنَا أُحْيِي وَأُمِيتُ ۖ قَالَ إِبْرَاهِيمُ فَإِنَّ اللَّهَ يَأْتِي بِالشَّمْسِ مِنَ الْمَشْرِقِ فَأْتِ بِهَا مِنَ الْمَغْرِبِ فَبُهِتَ الَّذِي كَفَرَ ۗ وَاللَّهُ لَا يَهْدِي الْقَوْمَ الظَّالِمِينَ (البقرة 258)

الآية الكريمة تتحدث عن تحدي ابراهيم عليه السلام للملك النمرود الذي ادعى الألوهية ، فقال له إبراهيم عليه السلام أن الشمس تطلع علينا من الشرق فاجعلها تطلع من الغرب !

وطلوع الشمس من الشرق هي حقيقة يشاهدها كل سكان الكرة الارضية، فهكذا تبدو الشمس لنا ، وكل لغات العالم تحتوي عن مصطلحي " شروق الشمس " و " غروب الشمس " ، وذلك لا يعني ان الشمس تدور حول الارض ، فالشروق والغروب يَحدُثان بسبب دوران الارض حول محورها، وبسبب دوران الأرض حول الشمس .

لا يخفى عليك، عزيزي القارئ انه من السخافة اتهام آيات القرآن بوجود اخطاء علمية !

رَبُّ الْمَشْرِقَيْنِ وَرَبُّ الْمَغْرِبَيْنِ

(الرحمن:17)

المقصود هو مَشرقَي الصيف والشتاء ، ومغربي الصيف والشتاء ، فمن المعروف علميًا أن زاوية طلوع أشعة الشمس تتغير في فصل الشتاء وفي فصل الصيف، وذلك بسبب التعديل الطفيف الذي يحدث في ميلان محور دوران الارض حول نفسها وينجم عن ذلك حدوث أقصر نهار ( أطول ليل ) وأطول نهار ( أقصر ليل )



للمزيد أقرأوا هنا


وَتَرَى الشَّمْسَ إِذَا طَلَعَت تَّزَاوَرُ عَن كَهْفِهِمْ ذَاتَ الْيَمِينِ وَإِذَا غَرَبَت تَّقْرِضُهُمْ ذَاتَ الشِّمَالِ وَهُمْ فِي فَجْوَةٍ مِّنْهُ ۚ ذَٰلِكَ مِنْ آيَاتِ اللَّهِ ۗ مَن يَهْدِ اللَّهُ فَهُوَ الْمُهْتَدِ ۖ وَمَن يُضْلِلْ فَلَن تَجِدَ لَهُ وَلِيًّا مُّرْشِدًا (الكهف 17)

الآية تخاطب الرسول (والقارئ) وتصف كهف الفتية بانه موجود بموقع بحيث يكون الكهف في ظل دائم وقت شروق الشمس ووقت غروبها ، ومرة أخرى الآية الكريمة لا تقول ان الشمس تدور حول الارض ، بل تقول ان الشمس تشرق وتغرب ( بالنسبة للواقف على سطح الأرض )، وهذه حقيقة نراها جمعينا .



وَسَخَّرَ الشَّمْسَ وَالْقَمَرَ ۖ كُلٌّ يَجْرِي لِأَجَلٍ مُّسَمًّى ۚ يُدَبِّرُ الْأَمْرَ يُفَصِّلُ الْآيَاتِ لَعَلَّكُم بِلِقَاءِ رَبِّكُمْ تُوقِنُونَ (الرعد 2)

الآية الكريمة تقول أن الشمس والقمر كل منهما يسبح في الفضاء ، وهذه أيضًا حقيقة علمية .

أستغرب كيف للمفتري ان يفترض ان الآية تقول ان الشمس تدور حول الارض !

وَالشَّمْسَ وَالْقَمَرَ ۖ كُلٌّ فِي فَلَكٍ يَسْبَحُونَ (الأنبياء 33)

نعم الشمس والقمر يجريان في الفضاء ، القمر يدور حول الارض والارض تدور حول الشمس, والشمس تجري بالفضاء وتدور داخل مجرتنا ( درب التبانه ) ، هذه حقائق علمية تتطابق مع آيات القرآن الكريم، تم إبرازها في الرد الأول

حَتَّىٰ إِذَا بَلَغَ مَغْرِبَ الشَّمْسِ وَجَدَهَا تَغْرُبُ فِي عَيْنٍ حَمِئَةٍ وَوَجَدَ عِندَهَا قَوْمًا ۗ قُلْنَا يَا ذَا الْقَرْنَيْنِ إِمَّا أَن تُعَذِّبَ وَإِمَّا أَن تَتَّخِذَ فِيهِمْ حُسْنًا (الكهف 86)

الآية تصف كيفية رؤية ذي القرنين لغروب الشمس، فبدت له الشمس تغرب داخل المستنقع الكبير ( عين حمئة ) ، بالضبط مثلما نذهب الى البحر ساعة الغروب فنرى الشمس كأنها تغطس داخل البحر !

الآية الكريمة تصف كيف يبدو المنظر بعين الناظر ولا يوجد هنا اي قول حول دوران مزعوم للشمس حول الأرض ، إلا داخل ذهن المفتري!

حَتَّىٰ إِذَا بَلَغَ مَطْلِعَ الشَّمْسِ وَجَدَهَا تَطْلُعُ عَلَىٰ قَوْمٍ لَّمْ نَجْعَل لَّهُم مِّن دُونِهَا سِتْرًا (الكهف 90)

نفس الأمر؛ للشمس مطلع وهو الشروق ولها أيضًا غروب. ومن الآية يُفهَم أن ذَا القرنين سار ليلًا ووافق مجيئه على قومٍ من الأقوام، ساعة شروق الشمس.

وهنا أيضًا تصف الآية الكريمة ما يراه الناظر على سطح الأرض، ولا يوجد اي تعارض بين الاية وبين دوران الأرض حول الشمس.

الشَّمْسُ وَالْقَمَرُ بِحُسْبَانٍ (الرحمن 5)

معنى الآية هو أن حركة كل من الشمس والقمر مضبوطة بدقة، وانه بحسب حركتهما نحسب التواريخ والأزمنة.

كيف وأين يفترض المفتري وجود أخطاء علمية !

وَسَخَّرَ لَكُمُ الشَّمْسَ وَالْقَمَرَ دَائِبَيْنِ وَسَخَّرَ لَكُمُ اللَّيْلَ وَالنَّهَارَ. (إبراهيم 33)

تذكر الآية الكريمة ان الله جعل الشمس والقمر في حركة دائمة من أجل حياة البشر . فالقمر يدور حول الارض والارض تدور حول الشمس, والشمس تسبح في الفضاء ضمن مجرة ( درب التبّانة ) . وقد يتساءل سائل ، لماذا ذكَرَت الآيات ايضًا ان الله ذلل الليل والنهار من اجل البشرية، الا يكفي ذِكِر الشمس والقمر ! والسبب هو، أنه من اجل تعاقب الليل والنهار باستمرار ( دائبين )، يجب ان تدور الارض حول نفسها - وهذه حقيقة علمية لم تكن البشرية تعرفها وقت نزول القران الكريم . ولذلك فَرَّقَت آيات الذكر الحكيم بين الشمس وبين النهار والليل.



أَلَمْ تَرَ إِلَىٰ رَبِّكَ كَيْفَ مَدَّ الظِّلَّ وَلَوْ شَاءَ لَجَعَلَهُ سَاكِنًا ثُمَّ جَعَلْنَا الشَّمْسَ عَلَيْهِ دَلِيلًا (الفرقان 45)

الآية فيها اشارة علمية خفية ، لأنها تفصل بين الشمس والظل ، والسبب هو ان الظل يحدث ايضًا بفضل دوران الارض حول محورها ، وهي حقيقة لم تكن معروفة للبشر وقت نزول القرآن الكريم. تقول الآية انه كان من الممكن ان يجعل الله الظل ساكنًا ،أي ان لا تدور الكرة الأرضية حول نفسها ، ولكن المشيئة الإلهية لم ترد أن يكون الظل ساكنًا فدارت الارض حول محورها ، وبسبب دوران الأرض فان أشعة الشمس تصل سطح الارض بزوايا تتغير خلال النهار الى حين غروب الشمس. إذًا يتحدد شكل الظل واتجاهه بحيل زاوية سقوط أشعة الشمس( ثُمَّ جَعَلْنَا الشَّمْسَ عَلَيْهِ دَلِيلًا ) .

لاحظ عزيزي القارئ كيف يحاول المفترون والحاقدون على الاسلام تشويه معاني آيات القرآن الكريم

عَنْ أَبِي ذَرٍّ رَضِيَ اللَّهُ عَنْهُ ، قَالَ : " قَالَ النَّبِيُّ صَلَّى اللهُ عَلَيْهِ وَسَلَّمَ لِأَبِي ذَرٍّ حِينَ غَرَبَتِ الشَّمْسُ : ( أَتَدْرِي أَيْنَ تَذْهَبُ ) ؟ قُلْتُ : اللَّهُ وَرَسُولُهُ أَعْلَمُ ، قَالَ : ( فَإِنَّهَا تَذْهَبُ حَتَّى تَسْجُدَ تَحْتَ العَرْشِ ، فَتَسْتَأْذِنَ فَيُؤْذَنُ لَهَا ، وَيُوشِكُ أَنْ تَسْجُدَ فَلاَ يُقْبَلَ مِنْهَا ، وَتَسْتَأْذِنَ فَلاَ يُؤْذَنَ لَهَا ، يُقَالُ لَهَا : ارْجِعِي مِنْ حَيْثُ جِئْتِ ، فَتَطْلُعُ مِنْ مَغْرِبِهَا ، فَذَلِكَ قَوْلُهُ تَعَالَى : ( وَالشَّمْسُ تَجْرِي لِمُسْتَقَرٍّ لَهَا ذَلِكَ تَقْدِيرُ العَزِيزِ العَلِيمِ )

يتحدث الحديث الشريف عن حركة الشمس في الفضاء ( وهو شيء صحيح كما بينت اعلاه ) وبين موقعها بالنسبة الى عرش الرحمن ، والعرش هو بناء ضخم وهو من امور الغيب . على أية حال ، الحديث الشريف بتاتا لا يتحدث عن دوران مزعوم للشمس حول الأرض

بالنسبة للحديث الشريف

"إنَّ مِن قِبلِ مغربِ الشَّمسِ بابًا مفتوحًا، عَرضُهُ سبعونَ سنةً، فلا يَزالُ ذلِكَ البابُ مفتوحًا للتَّوبةِ، حتَّى تطلعَ الشَّمسُ مِن نحوِهِ، فإذا طلَعت مِن نحوِهِ، لم ينفَع نفسًا إيمانُها، لم تَكُن آمنَت مِن قَبلُ، أو كسَبَتْ في إيمانِها خَيرًا"

فحوى الحديث يقول بأنه عند اقتراب قيام الساعة ودنو نهاية الحياة، سيأمر الله جل وعلا الشمس أن تطلع من الغرب وعندها يُدرك الناس أن يوم القيامة حق، بَيْد أنه آنذاك لن ينفع الكافر الندم . وطلوع الشمس من المغرب يعني انعكاس اتجاه دوران الأرض حول نفسها !

ومرة أخرى ، لا يتحدث الحديث الشريف عن حركة الشمس بالنسبة الى الأرض



الافتراء:

12 - الفشل بتحديات اله القرآن المحمّدي للبشر !

" يَا مَعْشَرَ الجِنِّ وَالإِنسِ إِنِ اسْتَطَعْتُمْ أَن تَنفُذُوا مِنْ أَقْطَارِ السَّمَوَاتِ وَالأَرْضِ فَانفُذُوا لاَ تَنفُذُونَ إِلاَّ بِسُلْطَانٍ . فَبِأَيِّ آلاءِ رَبِّكُمَا تُكَذِّبَانِ . يُرْسَلُ عَلَيْكُمَا شُوَاظٌ مِّن نَّارٍ وَنُحَاسٌ فَلاَ تَنتَصِرَانِ " (قصيدة الرحمن:33 ـ35)

التفسير

http://quran.v22v.net/tafseer-4934-55.html

اين هي شواظ النار التي ارسلت على روّاد الفضاء ولم انتصر اولئك و استطاعوا ان يخترقوا اقطار السماء !!؟؟؟

يحاول بعض البلهاء وشيوخ الدجل والكذب ايهام الناس ان البشر لم يطلعوا قط على سطح القمر ولم يخترقوا جدران الأرض او السماوات وهذه كذبة كبيرة و ما زال الكثير من رواد الفضاء يصعدون الى الفضاء سنوياً لعمل بعض الاصلاحات على بعض الأقمار الصناعيّة و التيلسكوبات

http://www.bbc.co.uk/science/space/universe/exploration/

http://www.buzzfeed.com/charliewarzel/a-tribute-to-the-universes-favorite-astronaut

امر مزعج جدا بعد كل ذلك التقدم العلمي ومجهودات العلماء والعباقرة الجبارة ان يقوم بعض من المسلمين بكل بلاهة بتكذيب امرهم ليتناسب مع تخاريف القرآن الغبي وتخاريفهم المحمدية الغبية الدموية التي اعتادوا عليها

-

المزيد من الفشل امام العلم ..

الرد :

كالعادة ، يشوه الحاقد على الاسلام معاني آيات الذكر الحكيم ، في محاولة منه لاتهامها بوجود أخطاء علمية ، والحقيقة هي العكس تماماً ، واليكم التوضيح :

يَا مَعْشَرَ الْجِنِّ وَالْإِنسِ إِنِ اسْتَطَعْتُمْ أَن تَنفُذُوا مِنْ أَقْطَارِ السَّمَاوَاتِ وَالْأَرْضِ فَانفُذُوا ۚ لَا تَنفُذُونَ إِلَّا بِسُلْطَانٍ (33) فَبِأَيِّ آلَاءِ رَبِّكُمَا تُكَذِّبَانِ (34) يُرْسَلُ عَلَيْكُمَا شُوَاظٌ مِّن نَّارٍ وَنُحَاسٌ فَلَا تَنتَصِرَانِ (35) [سورة الرحمن]

يخبرنا القران الكريم ان عدد السموات سبع، وأن كل سماء أكبر من سابقتها وتحيطها وتغلفها من كل الجهات "الَّذِي خَلَقَ سَبْعَ سَمَاوَاتٍ طِبَاقاً" (الملك : 3)

قطر السماء والأرض هو المسافة الواصلة من طرف السماء الى طرفها الآخر مرورًا بكوكب الأرض. وهي مسافة هائلة لا يتخيلها العقل!

يؤكد القرآن الكريم للبشر والجن بأنهم لن يصلوا إلى تلك المسافات إلا بقوة خارقة هائلة.

نلاحظ أن الوصول الى القمر أو إلى كوكبٍ آخر لا يعني النفاذ من قطر السموات والأرض. وكذلك ،الآية الكريمة لا تقول يإستحالة خروج الانسان الى الفضاء أو الهبوط على سطح القمر ، أو إرسال مُعدات من أجل استكشاف الفضاء!

بل إن القرآن الكريم يدعو الناس الى التفكر وإعمال البحث العلمي في نشأة الكون.

"وَيَتَفَكَّرُونَ فِي خَلْقِ السَّمَاوَاتِ وَالْأَرْضِ رَبَّنَا مَا خَلَقْتَ هَٰذَا بَاطِلًا"

﴿١٩١ آل عمران﴾

وتُخبر الاية 35 من سورة الرحمن، أن على فرض أن أيًا من الجن او الانس استطاع ان يقترب من نهاية أقطار السماء والأرض. فلن يتمكن من ذلك ، لانه سوف يُصاب بحُزم نارية من لهيب النحاس، والنحاس عنصر موجود في الفضاء، يتكون داخل لُب النجوم العملاقة قبل انفجارها وبعد الانفجار تُقذف غاز أيونات النحاس في الفضاء . وأليكم رابطًا يتحدث عن ذلك


ولذلك فمن الخطأ الفادح ، الادعاء بان القرآن الكريم يقول باستحالة خروج البشر الى الفضاء وأن آياته تتوعد رواد الفضاء برشقهم بنيران النحاس. فالوعيد بالآية موجه الى من يقترب الى نهاية السماء وهو شيء لم يحدث ومن غير الممكن حدوثه



الافتراء:

اللَّهُ يَعْلَمُ مَا تَحْمِلُ كُلُّ أُنثَىٰ وَمَا تَغِيضُ الْأَرْحَامُ وَمَا تَزْدَادُ ۖ وَكُلُّ شَيْءٍ عِندَهُ بِمِقْدَارٍ (الرعد :8) ما تزداد اي ما زادت الرحم في الحمل على ما غاضت حتى ولدته تماما

لا أحد يعلم ما تغيض الأرحام إلا الله ، ولا أحد يعلم متى يأتي المطر إلا الله حديث ثابت في الصحيحين

ماذا عن جهاز التصوير بالاشعة السينية - الايكو - ايها العالم بالغيب !!! و الذي يمكنه الآن تصوير الجنين و معرفة ادق التفاصيل في الرحم كقناتي فالوب ,.



الرد :

معنى الآية هو أن الله يعلم جنس الجنين وشكله وكل شيء عنه ، وكذلك يعلم مُسْبقاً مدة الحمل بالضبط ؛ هل سوف تزيد عن تسعة اشهر او (تغيض) تَقِّلْ عن ذلك. وهي أمور تعجز آلات المسح والفحوصات المخبرية عن معرفة موعدها المحدد مسبقًا



الافتراء:

ماذا عن علم واجهزة الأرصاد الجويّة الحديثة الذي يحدّد وقت هطول الأمطار وغالباً ما يصيب بتنبؤاته

- وَأَرْسَلْنَاْ الرِّيَاْحَ لَوَاْقِحَ فأسقيناكموه وما أنتم له بخازنين . معنى الآية الحقيقي يحتوي خطأ علمي فادح

تفسيرات القرطبي ابن كثير الطبري الجلالين ...... قالوا بأنن الرياح تلقّح السحاب فيما قال البعض الآخر من المفسرين بأن المقصود هو تلقح الغيوم و الشجر بالمياه ,فليست الرياح من تلقّح الغيوم إنّما تيّارات الهواء الناتجة عن تبخّر المياه المتصاعدة عن البحار - الرياح لا تلقّح الغيوم وهي ليست وحدها التي تقوم بتلقيح النباتات فهناك التلقيح الذي يتم بواسطة الحشرات والطيور أو يدويّاً من قبل الانسان و منها أنواع ما تقوم بالتلقيح الذاتي بنفسه

فيما ان الشق الثاني من الآية نقضه العلم الحديث اذ اصبح هناك ما يسمى بعملية الـ rainmaking حيث يتم وتلقيح الغيوم بشرياً بواسطة نثر أملاح و مواد كيميائية في الجو وانزال المطر بشريّاً

http://ar.wikipedia.org/wiki/مطر_اصطناعي

الرد:

وَأَرْسَلْنَا الرِّيَاحَ لَوَاقِحَ فَأَنزَلْنَا مِنَ السَّمَاءِ مَاءً فَأَسْقَيْنَاكُمُوهُ وَمَا أَنتُمْ لَهُ بِخَازِنِينَ (الحِجر:22)

تحتوي هذه الآية الكريمة على إشارات علمية، فهي تصف الرياح باللواقح، وتفيد بأن تلقيح الرياح للسحب بتسبب بإنزال المطر.

وتلقح الرياح السُحب بنويات التكاثف condensation nuclei وهي نويات تتجمع عليها جزيئات بخار الماء لتكون نُقطاً من الماء نامية داخل السحب، وتتكون نويات التكاثف من الماء، أملاح البحار والأتربة. ومن أجل أن تكون السحابة ممطرة، يلزم ان يكون لها مدد مستمر من بخار الماء ونوى التكاثف، المحمولة بواسطة الرياح أو الهواء الصاعد.

اما بالنسبة لادعاء المفتري على آيات القرآن بأن إمكانية إنزال المطر اصطناعيا تنقض معنى الآية، فهي محاولة خداع بائسة وفاشلة. فالآية تقول بأن الرياح مهمة لتلقيح الغيوم، وقد بينت ذلك أعلاه، وحتى تتم عملية إنزال المطر يجب أولًا ان تتواجد الغيوم! ومن أجل ذلك يلزم وجود الرياح! ونفس المصدر الذي ذكره المفتري على القرآن، يشرح بأن إنزال المطر يتم عن طريق نثر أملاح كيميائية من أجل جعل " الغيوم " ممطرة!

المطر الاصطناعي (بالإنجليزية: Artificial Rain) هو المطر الذي يتكوّن جراء عملية اصطناعية يقوم بها الإنسان عبر تلقيح الغيوم بأملاح ومواد كيميائية


لذلك حتى يكون ممن الممكن احداث مطر اصطناعي يجب ان تتواجد اولًا الغيوم ومن أجل ذلك يلزم وجود رياح!

لاحظوا كيف يكذب اعداء الاسلام، على القرآن الكريم، ويحاولون خداع الناس!

(وَاللَّهُ غَالِبٌ عَلَى أَمْرِهِ وَلَكِنَّ أَكْثَرَ النَّاسِ لَا يَعْلَمُونَ)



الافتراء:

13 - أخطاء و تناقضات أخرى

يقول القرآن ..

* ظَهَرَ الْفَسَادُ فِي الْبَرِّ وَالْبَحْرِ بِمَا كَسَبَتْ أَيْدِي النَّاسِ لِيُذِيقَهُمْ بَعْضَ الَّذِي عَمِلُوا لَعَلَّهُمْ يَرْجِعُونَ . الروم :41

قال المفسرون في الآية::

«ظهر الفساد في البر» أي القفار بقحط المطر وقلة النبات «والبحر» أي البلاد التي على الأنهار بقلة مائها «بما كسبت أيدي الناس» من المعاصي «ليذيقهم» بالياء والنون «بعض الذي عملوا» أي عقوبته «لعلهم يرجعون» يتوبون !!

في حين أنّ أغلب الدول الإسلاميّة تعاني مشاكل تصحّر وشح بالأمطار .. واكثرها شحّاً و قحوطا هم الأكثر تشدداً منهم على عكس دول الكفر و الفسق و الإلحاد والشرك بالله !!

الرد:

ظَهَرَ الْفَسَادُ فِي الْبَرِّ وَالْبَحْرِ بِمَا كَسَبَتْ أَيْدِي النَّاسِ لِيُذِيقَهُمْ بَعْضَ الَّذِي عَمِلُوا لَعَلَّهُمْ يَرْجِعُونَ﴿41﴾ قُلْ سِيرُوا فِي الْأَرْضِ فَانْظُرُوا كَيْفَ كَانَ عَاقِبَةُ الَّذِينَ مِنْ قَبْلُ كَانَ أَكْثَرُهُمْ مُشْرِكِينَ} [سورة الروم]

البر هو اليابسة، واليابسة ليست محصورة فقط في الصحاري, بل بالغابات والأدغال والمناطق الماطرة، والفساد في البر لا يعني فقط المجاعة والقحط . بل الأوبئة المستعصية، أزمات سياسية واقتصادية، حروب ، أورام خبيثة وكذا . لذلك فالفساد في البر ممكن ان يصيب كل الأقطار سواء كانت مسلمة ام غير مسلمة. ولأجل دحض افتراء المستلحد ،يكفينا ان نتذكر ظروف دول اوروبا الشرقية والاتحاد السوڤييتي سابقًا، الصعبة والقاسية، في حين تتمتع دولة تركيا بازدهار ورخاء متنامي

- الافتراء:

وَهُوَ الَّذِي مَرَجَ الْبَحْرَيْنِ هَذَا عَذْبٌ فُرَاتٌ وَهَذَا مِلْحٌ أُجَاجٌ وَجَعَلَ بَيْنَهُمَا بَرْزَخًا وَحِجْرًا مَحْجُورًا

تفسير الآية.: والله هو الذي خلط البحار: العذب السائغ الشراب، والملح الشديد الملوحة، وجعل بينهما حاجزًا من حجرٍ يمنع كل واحدٍ منهما من إفساد الآخر، ومانعًا مِن أن يصل أحدهما إلى الآخر

ان عدم اختلاط البحار هذا هو امر معروف منذ اول ما اخترع الانسان السفن البحرية وبدأ بالإبحار لكن الخطأ بالقول " وحجراً محجوراً "

والمفارقة الكبيرة في درجات الألوان ودرجات الملوحة بين البحار المتجاورة هو ما جعل البحّارة يكتشفون ذلك من غابر الأزمان فقببل اكثر من الف عام من الوجود المحمدي كتب عن ذلك أرسطو : راجع :

http://classics.mit.edu/Aristotle/meteorology.2.ii.html

http://books.google.com/books?id=9lV9voODu7UC&pg=PA32&lpg=PA32&dq=%E2%80%9CThe+drinkable,+sweet+water,+then,+is+light+and+is+all+of+it+drawn+up:+the+salt+water+is+heavy&source=bl&ots=uWtRLGnEgl&sig=2pa4yjd2LD_GKvgTTBz-WcJdkDI&hl=en&sa=X&ei=uVvPUYLoHsPaOfe5gDg&ved=0CC8Q6AEwAQ#v=onepage&q=%E2%80%9CThe%20drinkable%2C%20sweet%20water%2C%20then%2C%20is%20light%20and%20is%20all%20of%20it%20drawn%20up%3A%20the%20salt%20water%20is%20heavy&f=false

http://www.islam-watch.org/amarkhan/Miracles-of-Quran-Exposed.htm#15



الرد:

- القران الكريم هو أول من ذكر وجود حاجز بين مياه البحار والمحيطات المالحة وبين مياه البحار والمحيطات والأنهار العذبة.

والرابط الذي وضعه المفتري، ليس فيه ذكر لهذه الظاهرة. الرابط فيه مقال عن علم الأرصاد لأرسطو والذي كتبه سنة 350 قبل الميلاد، وفيه من الأخطاء العلمية ما فيه ، على أية حال ، يتحدث ارسطو في كتابه عن دورة المياه في الطبيعة ، ويقرر بان مياه الأنهار العذبة تتبخر ثم تتكثف وتعود الى الارض ، في حين تبقى مياه البخار المالحة (وهي اثقل وزنًا من المياه العذبة )في البحار ولا تتبخر. نحن نعرف أن كلام ارسطو غير دقيق لان مياه البحار تتبخر أيضًا، لكن ما يهمنا في هذا السياق هو ان أرسطو لم يذكر وجود حاجز يمنع اختلاط المياه العذبة بالمياه المالحة!

واليكم اقتباس من القسم الثاني في المقال part-2 الذي استشهد به صاحب الافتراء ، كي يوهم القارئ أن ارسطو سبق آيات القرآن في ذكر هذه الظاهرة



The drinkable, sweet water, then, is light and is all of it drawn up: the salt water is heavy and remains behind, but not in its natural place. For this is a question which has been sufficiently discussed (I mean about the natural place that water, like the other elements, must in reason have), and the answer is this. The place which we see the sea filling is not its natural place but that of water. It seems to belong to the sea because the weight of the salt water makes it remain there, while the sweet, drinkable water which is light is carried up.



كما تلاحظون، ارسطو لم يتحدث عن وجود فاصل يمنع اختلاط المياه العذبة بالمالحة، وانما اعتقد ان مياه الأنهار تتبخر واما مياه البحار وهي مالحة ، فلا تتبخر ، وللمزيد اليكم رابط الكتاب


أما بالنسبة للآية الكريمة " وَجَعَلَ بَيْنَهُمَا بَرْزَخًا وَحِجْرًا مَّحْجُورًا "

فهي اعجاز مذهل، فالآية ذكرت وجود حاجز ( برزخ )ومنطقة مفصولة ( حجرًا محجورًا ) عند التقاء النهر العظيم مع البحر ، ( من معاني البحر في اللغة، النهر العظيم ). تسمى منطقة التقاء النهر بالبحر ب "مَصَّب النهر" Estuary وهي المنطقة الانتقالية من النهر المحصور الى البحر المفتوح وتتصاعد نسبة الملوحة فيها من النهر باتجاه البحر. ولمصب النهر مميزات بيئية خاصه تجعله حيوياً لسلسلة غذاء اعداد ضخمه من الكائنات الحيه وبضمنها الانسان ، يفرغ النهر حمولته من الرواسب الطينية الغنية بالمواد العضوية وتفتات عليها الكائنات الدقيقة وتنمو فتصير غذاءً للمحار( الأصداف البحرية ) ولسرطانات البحر ( تقضي قسم من دورة حياتها في مصبات الانهار ) ولأنواع من الاسماك (مثل السلمون والرنجه ) وما سبق ذكره يصير مصدر غذاء لمستويات أعلى في السلسلة الغذائية كالأسماك الاكبر حجماً ،الطيور البحرية والبريه ( تتزود بالغذاء من مصاب الانهار إثناء هجرتها ) والثدييات ( دببه ، إنسان ..) ،ويوفر مصب النهر بيئة مثاليه لنمو العوالق النباتية " فيتو بلانكتون" والتي تضبط ( الى جانب النباتات والأشجار) نسبة ثاني اكسيد الكربون والأوكسجين في الطبيعة عن طريق عملية التمثيل الضوئي ، إضافة الى ذلك تحتضن بيئة مصبات الأنهار يرقات لبعض أنواع الأسماك البحرية وهي مصدر ل%87 من المأكولات البحرية في أمريكا ولمصبات الانهار دور رئيسي في ضبط جودة المياه المسكوبة في البحار إذ تنمو في قعر مصبات الانهار وعلى ضفافها مملكة نباتيه مميزه تقوم بتنقية وتحليل ( الى حد كبير ) السموم والملوثات المفرزة في مياه الانهار من الملوثات ونفايات الصناعة قبل سريانها الى البحر.

كانت مصبات الانهار وما زالت من أكثر النظم البيئية خصوبةً وحيوية لحياة البشر ويكفي ان نعلم ان 60 ٪ من سكان المعمورة يقطنون في مدن منتشرة على ضفاف مصبات الانهار .

وعلى سبيل المثال : نيويورك ، كالفورنيا ، سان فرانسيسكو، جاكرتا .

ان وجود منطقه فاصله بين المياه العذبة والمالحة ( حجراً محجورا) يؤثر بشكل مباشر أو غير مباشر على حياة وبقاء جميع انواع الكائنات الحيه ولذلك تنشط في الغرب حركات حماية البيئة الداعية الى وقف اتلاف بيئة مصاب الانهار بسبب تراكم النفايات فيها والعمل على اصلاحها والحفاظ عليها كمحميات طبيعية.

فقط في عصرنا الحالي، اكتشفت الميزات البيئية الخاصة لمنطقة مصب الأنهار ، في حين ذكر القران الكريم وجود تلك المناطق والتي لا تُرى بالضرورة بالعين ، لأنه لا يُرى في كل مصب نهر، اختلاف بألوان المياه الاختلاف بالملوحة بين مياه البحر والنهر



المصادر






الافتراء:

العلم يكذّب أن يكون القمر انشق يوما !

http://ibiscience.com/?p=533

http://youtu.be/wBZ-DYag-wU

http://lunarscience.nasa.gov/?question=evidence-moon-having-been-split-two

. الرد:

بالنسبة لمعجزة انشقاق القمر ، فالحقائق تدعم حدوث المعجزة، بداية ، نحن نؤمن بالواقعة لان القرآن - معجزة الله الخالدة يخبرنا بذلك ولأننا نصدق كلام رسول الحق .

وعلى أية حال، من يبحث عن دليل علمي ، فإليكم آخر ما توصل اليه علماء ناسا.

فقد تبين أن سطح القمر مشقق وفيه شقوق عميقة تصل الى عمق 20 كيلومترًا ،وتمتد الشقوق الى 300 كيلومترًا ويتراوح عرض الشق المتر الواحد!

وللمزيد من المعلومات اقرأوا هذا المصدر


إن من شأن هذه المعطيات المكتشفة ان تكون إشارة علمية قوية على حدوث انشقاق للقمر في الازمنة الماضية





الافتراء:

ولم يثبت العلم أيضاً وجود حشرة تعيش على ظهر البعوضة !! ولا أنَّ النمل فيه زجاج !!! كما يدّعي المدلّسين

http://new.elfagr.org/Detail.aspx?nwsId=214126&secid=43&vid=2#.UYRR9n55SKA

الرد:

بالنسبة للآية الكريمة التي تذكر بعوضة وما فوق البعوضة

إِنَّ اللَّهَ لَا يَسْتَحْيِي أَن يَضْرِبَ مَثَلًا مَّا بَعُوضَةً فَمَا فَوْقَهَا ۚ فَأَمَّا الَّذِينَ آمَنُوا فَيَعْلَمُونَ أَنَّهُ الْحَقُّ مِن رَّبِّهِمْ ۖ

(البقرة 26)

فقد اعتبر الأئمة المفسرين – رحمهم الله جميعا ً– ان لكلمة ( فوق ) معنيان

1- الزيادة في الحجم، أي أن الله يضرب المثل بالبعوضة و بما هو أكبر منها حجما ً.

2- الزيادة في الوصف، أن الله يضرب المثل بالبعوضة بالصغر وضآلة الحجم و بما هو أقل منها.

و لكن حديثا ًو بعد اختراع المجاهر الإلكترونية ، يتضح بأن هذه الكلمة عبارة عن ظرف مكان ، اي انها تعني ايضًا وجود مخلوقات تسكن على جسم البعوضة. فقد تم اكتشاف كائنات حية دقيقة تعيش على ظهر البعوضة، منها انواع من ذباب صغير يسمى midge

واليكم مقالًا يتحدث عن ذلك


إن في الإشارة الى و جود هذه الكائنات الحية إعجاز فمن كان يتخيل وجود مثل هذه الكائنات الحية في زمن الرسول عليه الصلاة والسلام ، و كيف يجرؤ أحد على قول ذلك و هي لا ترى إلا ّ بالمجهر الإلكتروني !

بالنسبة لمبنى النمل ، فالقرآن الكريم لا يقول ان في النمل زجاج ! فهذا افتراء صارخ من المستلحد.

الآية الكريمة تذكر ان نملة طلبت من بقية النمل الاختباء ، خوفًا من ان يتحطموا تحت أرجل جنود سليمان عليه السلام.

( يَا أَيُّهَا النَّمْلُ ادْخُلُوا مَسَاكِنَكُمْ لَا يَحْطِمَنَّكُمْ سُلَيْمَانُ وَجُنُودُهُ وَهُمْ لَا يَشْعُرُونَ )

ومن معاني كلمة (تحطيم) هو التكسير، وفي ذلك إشارة خفية الى وجود خصائص زجاجية في مبنى النمل لكن ذلك لا يعني ان النمل مصنوع من زجاج !

فالنمل لا يوجد له هيكل عظمي بل يغلّفه هيكل خارجي وهو عباره عن طبقه صلبه عازله للماء تحمي الجسم وتحفظ بنيته. تنبع صلابة الهيكل الخارجي من مادة ال Chitin المتكونة من سلاسل ( بوليمرات )من المواد السكرية ذات مبنى بلوري مرزومه بألياف دقيقه وتدخل مادة ال chitin في تكوين (المفصليات ) مثل النمل الخنافس وأم اربع واربعين ، والحشرات الطائرة مثل الفراشات النحل والجنادب وفِي الكائنات البحرية كسرطان البحر والچمبري. إضافة لذلك يستعمل ال chitin في صناعة الاغذية والورق وغيرها. في 2013 أعلن عن اكتشاف نوع غريب من النمل أبيض اللون ويبرز اللون الأبيض الناصع المبنى الزجاجي القابل للكسر والتهشيم.

يعيش هذا النمل الأبيض في غابات الفليبين والذكور منه بيضاء بالكامل وأما الاناث فيكسو كل عين من عينيها شريط بني وأصفر بشكل مشابه للقراصنة ولذلك سمي ذلك النوع ب(النمل القرصان ) Cardiocondyla pirata.

واليكم المصدر، والذي يحتوي على صور مذهلة لمبنى النمل الزجاجي الخواص


الافتراء:

وبغض النظر عن محاولات التلاعب من قبل بعض المتحاذقين المسلمين بان ببول البعير أدوية يمكن استخلاصها بعد التخلص من مادة البولينا السامة التي فيه لتقوية الشعر و كفوائد للبشرة ..... الخ لم يتجرّاً اي احد منهم نفي قدرته على التسبب بتلف كلوي للإنسان !!

و لا بإثبات علمي لأي فائدة طبيّة للحجامة أو الحبّة السوداء و ماء التين او وجود اي مضاد سمّي أو أي قدرة للتمر او العجوة على علاج التسمم الذين وردت بهم احاديث ثابتة في الصحيحين !!

فوفقاً لجمعية السرطان الأمريكية، "الدليل العلمي الموجود لا يدعم الحجامة كعلاج للسرطان أو أي مرض آخر

https://ar.wikipedia.org/wiki/حجامة

http://www.cancer.org/treatment/treatmentsandsideeffects/complementaryandalternativemedicine/herbsvitaminsandminerals/cupping

بول البعير و التلف الكلوي الحتمي طبيّاً

http://www.youm7.com/News.asp?NewsID=797763



الرد:

بالنسبة لبول البعير او الابل، فالمستلحد يتهجم ويفتري متجاهلًا كل الحقائق العلمية والتي تتطابق مع ما ورد في السنة النبوية الشريفة.

ورد في الحديث الشريف ان بول الابل يحتوي على خصائص علاجية للتَّداوي ففي "الصحيحَيْنِ" وغيْرِهما من حديثِ أنَسِ بْنِ مالكٍ قال: "قدِمَ رهْطٌ من عُرينةَ وعُكْلٍ على النَّبيِّ صلى الله عليه وسلَّم فاجْتَوَوُا المدينة، فشَكَوْا ذلك فَأَمَرَهُمْ أَنْ يَأْتُوا إِبِلَ الصَّدَقَةِ، فَيَشْرَبُوا مِنْ أَبْوَالِهَا وَأَلْبَانِهَا، فَفَعَلُوا فَصَحُّوا وَسَمِنُوا"، وعند النَّسائي: "...حتَّى اصفرَّتْ ألوانُهم وعظُمَتْ بطونُهم".

وطبعًا قبل اكثر من 1440 سنة، لم يكن بالإمكان الاستفادة من الخصائص العلاجية الموجودة في بول الابل الا بشربها، واليوم تُستَخلَص الميزات العلاجية دون اللزوم لشرب بول الابل. ولمن ينكر وجود خصائص علاجية لبول الابل، فإليه هذا الرابط من محرك البحث للأبحاث الطبية pubmed، مَنْ يتصفح الرابط، يجد عشرات الابحاث التي تنسب خصائص مضادة للسرطان ولأمراض اخرى، موجودة في بول الابل






الحبة السوداء

روى البخاري ومسلم في كتاب الطب، وابن ماجه وأحمد وغيرهم من حديث أبي هريرة قال: قال رسول الله صلى الله عليه وسلم: الحبة السوداء شفاء من كل داء إلا السام. أي الموت.

والحبة السوداء واسمها العلمي Nigella Sativa، تُجرى على فوائدها الطبية أبحاث كثيرة في تقوية جهاز المناعة، وخواصها المضادة للجراثيم والسرطان، وتخفيف التهابات المفاصل وغيرها، واليكم رابط فيه نبذة من تلك الابحاث




الحجامة

أما الحجامة ،فهي إخراج الدم الفاسد من الجسم بواسطة كاسات الحجامة والتي تسحب وتمص الدم، والحجامة سنةٌ نبويةٌ، فقد وردت أحاديث صحيحة عديدة في فضل الحجامة. فعن أنس قال: قال رسول الله صلى الله عليه وسلم: "إن أمثل ما تداويتم به الحجامة"، وعن أبي هريرة قال: قال رسول الله صلى الله عليه وسلم: "إن كان في شيء مما تداوون به خير ففي الحجامة"



ومعروف ان الحجامة تُسَّرِع تعافي عضلات الجسم من التشنجات والانقباضات، وقد ظهرت على جسم السباح الأمريكي صاحب ال 8 ميداليات ذهبية " مايكل فيلبس " في أولمبياد 2016، علامات حجامة وحين سُئل عنها ، قال انها قام بالحجامة حتى يريح عضلاته المتشنجة، واليكم المصدر


طَرَقَتْ الحجامة أبواب الطب البديل في الغرب منذ سنوات قليلة، لذلك وحتى هذه الساعة (2019)، هناك حاجة لإجراء دراسات علمية أكاديمية حول فوائد الحجامة. وذلك من اجل تاكيد نتائج الأبحاث الأولية التي تتحدث عن مساهمة الحجامة في تخفيف حدة آلام أسفل الظهر، الصداع والآن العضلات

اليكم ملخصها


ونحن واثقون من صحة كلام رسول الحق، لأنه ينطق بإلهامٍ من الله، فكما صدق حديثه بخصوص فوائد بول الابل والحبة السوداء، فصادق حديثه بصدد نجاعة الحجامة والدليل هو إقبال الرياضيين من الغرب على ممارسة الحجامة.

التمر

بالنسبة للتمر فقد قال رسول الله صلى الله عليه وسلم (من تصبح بسبع تمرات عجوة لا يصيبه في هذا اليوم سُمّ ولا سِحر) صحيح البخاري.



والحديث الشريف ، اليكم هذه الدراسة والتي تثبت فعالية مستخلص حبة التمر ضد سموم جرثومة Streptococcus pyogenes

اليكم رابط الدراسة




واليكم دراسة اخرى،

بعنوان : " دراسة تأثير خلاصة التمر على إبطال مفعول سم الحية والعقرب "، خلاصة الدراسة تقول أنه : " تم إعطاء أربعة متبرعين من (9 - 11) حبة تمر لكل منهم ، أما عينات الدم فتم أخذها قبل أكل التمر وبعده بحوالي (4 - 5) ساعات ، فكشفت الدراسة أن عينات الدم التي أخذت منهم بعد تناول التمر كانت مقاومة لسم الأفعى بنسبة (83%) ، وأن نسبة امتصاص الهيموغلوبين لسم الأفعى وتأثيره على (3%) من خلايا الدم الحمراء قبل تناول التمر كانت (0.542) ، وبعد تناول التمر أصبحت (0.09)، وقد وجدت الدراسة أو التجربة أن إعطاء (5%) من خلاصة التمر أبطلت حوالي (34%) و (71%) من النشاط السمي للأفعى والعقرب على التوالي، وأن (20%) من خلاصة التمر أحبطت المفعول بنسب (87%) و (100%) "



وللنظر في تفاصيل هذه الدراسة يمكن مراجعة المرجع التالي




واليكم دراسة اخرى، تظهر تسبب مستخلص حبات التمر بموت خلايا سرطانية في الكبد hepatocellular carcinoma



اليكم رابط الدراسة




مما ذكر اعلاه نزداد يقينًا ان رسول الحق لا ينطق عن الهوى وانه حين يأمر بشيء او ينهى عنه فانه يتحدث بالهامٍ من الله.



اللهم صَل وسلم وبارك على محمد وعلى آله وصحبه وسلم





الافتراء:

14 - التناقضات التاريخية بالقرآن !!

ـ يقول القرآن: في سورة طه 85 ـ 88

 "قَالَ فَإِنَّا قَدْ فَتَنَّا قَوْمَكَ مِنْ بَعْدِكَ وَأَضَلَّهُمُ السَّامِرِيُّ فَرَجَعَ مُوسَى إِلَى قَوْمِهِ غَضْبَانَ .... فَكَذَلِكَ أَلْقَى السَّامِرِيُّ فَأَخْرَجَ لَهُمْ عِجْلاً جَسَدًا لَهُ خُوَارٌ فَقَالُوا هَذَا إِلَهُكُمْ وَإِلَهُ مُوسَى فَنَسِيَ"



حسب الروايات التوراتيّة و الأبحاث التاريخيّة تثبت لنّا أن قصّة موسى قد ظهرت في الفترة ما بين 1500 - 1400 ق.م

و السامري الذي صنع العجل هو اللقب الذي كان ينسب لمن كان يعيش بالسامرة - المدينة الإسرائيلية - التي كانت قد بنيت عام 880 ق.م أي بعد ظهور اسطورة موسي بحوالي سبعة قرون و القصة هذه ورد ذكرها بالتوراة اليهوديّة مع قصّص اخرى للسامريين , خلط بينهما من اقتبس قصصهم فالتوراة كان تحديدها للتواريخ اكثر دقّة من القرآن المقتبس منها الذي وقع مؤلّفه الآخذ عنها حكاياه و اساطبره في مآزق التناقضات التاريخيّة بسبب عدم درايته الجيدة بدينهم

وحسب المؤرخين و البحوثات الآثريّة في مئات الألواح المكتشفة والرسومات والوثائق التي تتحدث بالتفصيل عن تاريخ المصريّين القدماء وملوكها وعلاقاتها الخارجية وحروبها مع الدول المجاورة لم يوجد في وثائقه أي آثر لشخص يدعى موسى، كما لا يوجد أي ذكر او إشارة أو اي وثائق او أثر لأتباع او صحابة، لنبي أو أمير أو قائد باسم موسى النبي ،و لصبي اسرائيلي تبنته ابنة فرعون وربي في القصر الفرعوني . كما لا يوجد أي ذكر لفرعون غرق وجيشه في البحر .



- فلم تنجح اقتباسات محمد من التوراة و آيات اناجيل المسيحيّين في النجاة من الأخطاء التاريخية



الرد:

الاسم ( سامريّ) كان موجودًا قبل بناء مدينة السامرة، والدليل موجود في التوراة التي يزعم صاحب الافتراء أن القرآن منقول عنها.

تذكر التوراة أنَّ باني مدينة ( السامرة ) واسمه ( عُمري )، سماها بهذا الاسم على اسم الشخص أو القبيلة التي باعت الأرض.

واليكم نَصّ الاقتباس من التوراة بالعربية والإنجليزية.

سفر ملوك الاول إصحاح 16

23 فِي السَّنَةِ الْوَاحِدَةِ وَالثَّلاَثِينَ لآسَا مَلِكِ يَهُوذَا، مَلَكَ عُمْرِي عَلَى إِسْرَائِيلَ اثْنَتَيْ عَشَرَةَ سَنَةً. مَلَكَ فِي تِرْصَةَ سِتَّ سِنِينَ.

24 وَاشْتَرَى جَبَلَ السَّامِرَةِ مِنْ شَامِرَ بِوَزْنَتَيْنِ مِنَ الْفِضَّةِ، وَبَنَى عَلَى الْجَبَلِ. وَدَعَا اسْمَ الْمَدِينَةِ الَّتِي بَنَاهَا بِاسْمِ شَامِرَ صَاحِبِ الْجَبَلِ «السَّامِرَةَ».

In the thirty-first year of Asa the king of Judah, Omri ruled over Israel twelve years-in Tirzah, he ruled six years.23



And he bought the hill Samaria of Shemer for two talents of silver; and he built on the hill, and called the name of the city which he built, after the name of Shemer, the owner of the hill, Samaria.24

اذًا كما ترى، عزيزي القارئ، الاسم كان متعارفًا عليه بين تلك الأقوام، قبل إنشاء مدينة السامرة.

والحقيقة، أن قصة السامري في القرآن الكريم، هي أحد الإثباتات الصارخة بأن القرآن مصدره من السماء. فالقرآن ينسب فكرة صُنع العجل الذهبي الى السامري، بينما تنسب التوراة ذلك الفعل الشنيع الى هارون شقيق موسى!

اقرأ عزيزي القارئ ما تذكره التوراة

1 ولما رأى الشعب ان موسى ابطأ في النزول من الجبل اجتمع الشعب على هرون. وقالوا له قم اصنع لنا آلهة تسير امامنا. لان هذا موسى الرجل الذي اصعدنا من ارض مصر لا نعلم ماذا اصابه.

2 فقال لهم هرون انزعوا اقراط الذهب التي في آذان نسائكم وبنيكم وبناتكم وأتوني بها.

3 فنزع كل الشعب اقراط الذهب التي في آذانهم وأتوا بها الى هرون.

4 فاخذ ذلك من ايديهم وصوّره بالازميل وصنعه عجلا مسبوكا. فقالوا هذه آلهتك يا اسرائيل التي اصعدتك من ارض مصر.

(سفر الخروج ، إصحاح 32)

فَلَو كان القرآن منقولًا عن التوراة (على فرض) لاتهمت آيات القرآن هارون وليس السامري!

وَمَا كَيْدُ الْكَافِرِينَ إِلَّا فِي ضَلَالٍ!

بالنسبة لباقي التساؤلات التي طرحها المستلحد حول توثيق تاريخي لقصة النبي موسى، فالدليل من ناحيتنا هو القرآن الكريم، فنحن لسنا بحاجة الى دلائل أثرية حتى نُصَدق قصص القرآن،

القران كتاب محفوظ من عند الله، ومن يشكك في ذلك فليتفضل وليثبت لنا تشكيكه وليكتب لنا بكلماته كتابًا يضاهي القرآن الكريم، فليتفضل!



الافتراء:

: تناقضات ذكر مريم - أم المسيح " عيسى "

ـ ورد في سورة التحريم -آية : 66 " وَمَرْيَمَ ابْنَتَ عِمْرَانَ الَّتِي أَحْصَنَتْ فَرْجَهَا فَنَفَخْنَا فِيهِ مِنْ رُوحِنَا وَصَدَّقَتْ بِكَلِمَاتِ رَبِّهَا وَكُتُبِهِ وَكَانَتْ مِنَ الْقَانِتِينَ"

ـ وفي سورة مريم آيات : 19- 27 - 28 " فَأَتَتْ بِهِ قَوْمَهَا تَحْمِلُهُ قَالُوا يَا مَرْيَمُ لَقَدْ جِئْتِ شَيْئًا، يَا أُخْتَ هَارُونَ مَا كَانَ أَبُوكِ امْرَأَ سَوْءٍ وَمَا كَانَتْ أُمُّكِ بَغِيَّا"

- الخطأ الذي وقع به كاتب القرآن هنا أن هاروون الذي هو اخو موسي عاش 1500 عام !!!!! وهو الفارق بين وجود خرافة موسي تاريخياً و مريم ام المسيح !!

والقرآن يذكر أن مريم - أم المسيح - هي بنت عمران وأخت هارون أخو موسى، و انجيل المسيحيين و الوثائق التاريخيّة المدوّنة توضح أن أم المسيح - مريم - أبوها كان يدعى " هالي " و لم يكن لها إخوة !!.

وهذا الالتباس حدث بسبب سوء الاقتباس من قبل مقتبس القصص اليهودية والانجيلية محمد فقد ذكرت الكتب اليهودية قصّة المطابقة لكلامه ذلك والتي اقتبسها منها ذلك البدوي وهي قصّة " عمرام " والد هارون حيث كان له بنت تدعى مريم / فَوَلدَتْ لِعَمْرَامَ هَارُونَ وَمُوسَى وَمَرْيَمَ أُخْتَهُمَا" (العَدَدِ 26: 59؛ الخروج 15: 20) /

ولكنها لم تكن أم المسيح بل كانت نبية (سفر أخبار الأيام الأول 6: 3) و ذكرت أيضاً تلك الأسفار أن مريم ابنة عمرام هذه كانت تزوجت برجل يدعى حور وأنجبت منه ناداب وأبيهو وذلك سنة 1500 ق.م .

الرد:

كلمات القرآن مُنّزلة من عند الله ولذلك فهي دقيقة ومضبوطة. الاية ذكرت أنَّ قوم مريم عليها السلام، خاطبوها بكلمة (أخت ) وليس ( شقيقة) وفِي ذلك فرق كبير , فالشقيق هو الأخ من الأب والأم، أما الاخ فيمكن أن تُطلق من باب نَسْبْ الأشخاص الى أُناس يشابهونهم في الصفات والمزايا، وهو ما قصده قوم مريم عليها السلام بخطابهم لمريم ، وقد سُئِلَ أحد الصحابة عن سبب مخاطبة مريم باخت هارون فسأل الرسول صلى الله عليه وسلم فأجابه صلى الله عليه وسلم بان الأقوام السابقه كانت تنسب أهل الصلاح الى الأنبياء السابقين والرجال الصالحين ، واليكم نَصَّ الحديث مما

رَوَاهُ مُسْلِمٌ رَحِمَهُ اللَّهُ تَعَالَى فِي صَحِيحِهِ [2135] عَنِ الْمُغِيرَةِ بْنِ شُعْبَةَ قَالَ : لَمَّا قَدِمْتُ نَجْرَانَ سَأَلُونِي فَقَالُوا : إِنَّكُمْ تَقْرَءُونَ : يَا أُخْتَ هَارُونَ ، وَمُوسَى قَبْلُ عِيسَى بِكَذَا وَكَذَا ؟

فَلَمَّا قَدِمْتُ عَلَى رَسُولِ اللَّهِ صَلَّى اللَّهُ عَلَيْهِ وَسَلَّمَ سَأَلْتُهُ عَنْ ذَلِكَ ، فَقَالَ : إِنَّهُمْ كَانُوا يُسَمُّونَ بِأَنْبِيَائِهِمْ وَالصَّالِحِينَ قَبْلَهُمْ اهـ ..

وبما أن بين مريم عليها السلام وبين هارون عليه السلام زمنًا طويلًا

فالمقصود رَجُلٌ صالح يحمل نفس الاسم هارون ، وليس هَارُونَ أخ مُوسَى عليهما السلام.





الافتراء:

التناقض في وجود عقوبة الصلب.

يخبر القرآن أن فرعون الذي حكم مصر في زمن الفراعنة قال : "لأُقَطِّعَنَّ أَيْدِيَكُمْ وَأَرْجُلَكُم مِّنْ خِلاَفٍ ثُمَّ لأُصَلِّبَنَّكُمْ أَجْمَعِينَ." ( الأعراف 7: 124 _ الشعراء 26: 49 )

في حين أن الدلائل التاريخية تؤكّد أن أول من اخترع عقوبة الصلب وقام بها هم الرومان القدماء قبل 700عام من وجود المسيح اي بعد اندثار الفراعنة بمئات السنين !! و أن الفراعنة لم يكونوا قد عرفوا او استخدموا الصلب قط

http://en.wikipedia.org/wiki/Crucifixion



الرد:

- عملية الصَلْبْ كانت معروفة لشعوب سَبَقت الرومان، ومن بينهم مصر القديمة في فترات حُكم الفراعنة، وآثار الفراعنة تشتمل على رسومات للصليب

في هذا الرابط تجدون صور من آثار الفراعنة تتضمن شعار الصليب


وذكرت أساطير مصر القديمة قصص صَلْب، لآلهتهم (اوزيريس ) و(حورس)

اليكم اقتباس مع تَرجمته من كتاب

Thomas W. Doane, Bible Myths and Their Parallels in Other Religions (484) صفحة

"Osiris, the Egyptian Saviour, was crucified in the heavens. To the Egyptian the cross was the symbol of immortality, an emblem of the Sun, and the god himself was crucified to the tree, which denoted his fructifying power.

"Horus was also crucified in the heavens. He was represented, like...Christ Jesus, with outstretched arms in the vault of heaven."

" (أوزيريس) المُخلصّة المصرية ، صُلبت في السموات. إعتَبَر المصريون (القُدامى) الصليب رمزًا للخلود وشعارًا يَدُل على الشمس، والإله ذاته كان مصلوبًا على الشجرة والتي تُعَّبِر عن شدة خُصوبته"

" وكذلك( حورَس) صُلِبَ في الجنّة. تمَّ رَسْمَهُ مصلوبًا في أعلى الجنة، ذراعاه مفتوحتان حتى آخرهما مثل عيسى المسيح

وهذا هو رابط الكتاب من صفحة رقم 484


وطبعًا، نحن لسنا بحاجة الى تأكيدات تاريخية من أجل نُصَدق كلام الله عز وجل!

-



الافتراء:

- التناقض التاريخي في قضيّة استعمال عملة " الدراهم "

- ذكر القرآن أن الأعراب اشتروا يوسف بثمن بخس بدراهم معدودة / سورة يوسف : 20

و الخطأ الموجود أن الدراهم لم تكن موجودا أيام يوسف و لا حتى موسى الذي ظهر بعده .., لأن الدرهم أول من استعمله هم اليونانيون (1100 ق.م ) و كان اسمه (درخمة) وعنهم أخذ الرومان العملة تلك ؛ وعندما جاء عصر محمد كان تجّار العرب يتعاملون مع الروم ؛ فنشروا تلك العملة الرمانية بأراضيهم ؛ الى أن جاء عصر عبد الماك بن مروان الأموي وسك الدرهم والدينار العربي ؛ أما العملة التي اشتري بها يوسف ففد كانت الفضة !!.

مراجع :

http://en.wikipedia.org/wiki/--dir--ham

http://en.wikipedia.org/wiki/Drachma

http://thawra.alwehda.gov.sy/_--print--_veiw.asp?FileName=61304550320080515220347







الرد:

كانت العملات المعدنية معروفة في مصر القديمة، ولكنها لم تكن مصكوكة اي لم يكن يدُفَع ويُشترى بها ، بل كانت عبارة عن وحدات وزن معيارية من الفضة والنحاس تُستعمل لتحديد قيمة الغَرض ، اليكم هذا الاقتباس مع ترجمته

Before they started using coins around 500 B.C., Egyptians used a system of value based on the weights of various metals, especially silver and copper. According to the British Museum, records from the 18th dynasty (1550-1295 B.C.) reveal that the metals themselves were not exchanged between people at that time even though they were used to determine the value of otherthings.

قبل انتشار العملات المعدنية حوالي 500 ق . م استخدم المصريين القدماء في البيع والشراء داخل أسواقهم نظام المقايضة أي تبادل البضائع وكانوا يثمنون قيمة تلك البضائع التي سوف يتم تبادلها عن طريق أوزان المعادن وخاصةً الفضة والنحاس. تُظهر سجلات المتحف البريطاني والتي تعود الى الأسرة الثامنة عشر ( 1295-1550) ق.م. إنَّ المعادن لم تكن مُتبادَلَة في العمليات التجارية ، على الرغم من أنها كانت تُستعمل لتحديد قيمة الأغراض.

وللمزيد من المعلومات اليكم الرابط


بالنسبة لمصطلح درهم فليس مؤكدًا أن الاسم منقول عن اليونانيين وانما هو افتراض بسبب تشابه اللفظ، ويمكن ان يكون الاسم منقولًا من العملة الفارسية القديمة (دْراهم ) drahm ، للمزيد انظروا هنا


وعلى اية حال ، فالادعاء أنَّ العملات النقدية لم تكن قائمة زمن يوسف عليه السلام ، هُو ادعاء خاطئ فكما رأينا كانت موجودة وكانت تستعمل لتقدير السلع وتثمينها ، ولم يكن لزامًا الدفع والشراء بها، والقرآن الكريم أخبرنا أنهم باعوه بثمن بخس أي أنه كان يتحدث عن تثمينه و تقدير قيمته بدراهم معدودة وقد تكون هناك عملة (وزن )مصرية او غير مصرية كانت تُسمى "درهم " آنذاك ،ولم تُكتَشف حتى يومنا هذا، أو يكون قصد الآية الشريفة ان الثمن الذي دُفِع مقابل يوسف كان مساويًا لبضعة دراهم المعروفة لأهل الجزيرة العربية ،وقت نزول الآيات الكريمة

قال الله تعالى :- (وَشَرَوْهُ بِثَمَنٍ بَخْسٍ دَرَاهِمَ مَعْدُودَةٍ وَكَانُواْ فِيهِ مِنَ الزَّاهِدِينَ (20) وَقَالَ الَّذِي اشْتَرَاهُ مِن مِّصْرَ لاِمْرَأَتِهِ أَكْرِمِي مَثْوَاهُ عَسَى أَن يَنفَعَنَا أَوْ نَتَّخِذَهُ وَلَدًا وَكَذَلِكَ مَكَّنَّا لِيُوسُفَ فِي الأَرْضِ وَلِنُعَلِّمَهُ مِن تَأْوِيلِ الأَحَادِيثِ وَاللَّهُ غَالِبٌ عَلَى أَمْرِهِ وَلَكِنَّ أَكْثَرَ النَّاسِ لاَ يَعْلَمُونَ (21)) (سورة يوسف)





الافتراء:

- قال القرآن :

وَقَالَتِ الْيَهُودُ عُزَيْرٌ ابْنُ اللَّهِ وَقَالَتِ النَّصَارَى الْمَسِيحُ ابْنُ اللَّهِ ۖ-;---;-- ذَٰ-;---;--لِكَ قَوْلُهُمْ بِأَفْوَاهِهِمْ ۖ-;---;-- يُضَاهِئُونَ قَوْلَ الَّذِينَ كَفَرُوا مِنْ قَبْلُ ۚ-;---;-- قَاتَلَهُمُ اللَّهُ ۚ-;---;-- أَنَّىٰ-;---;-- يُؤْفَكُونَ ﴿-;---;--30﴾-;---;--

لا يوجد أي نص توراتي أو يهودي يشير الى أن عزرا أو "عزير" هو ابن الله !!! , و لم يثبت تاريخياً وجود أي تحريف بالتوراة عبر الزمان أو حتى وجود طائفة يهوديّة كانت تعبد من يدعى عزير ..

https://ar.wikipedia.org/wiki/عزرا

http://ar.wikipedia.org/wiki/مخطوطات_الكتاب_المقدس

http://ar.wikipedia.org/wiki/مخطوطات_البحر_الميت



الرد:

العزير الذي قالت عنه اليهود أنه ابن الله المقصود به ، عزرا بن يوسف او عزرا الكاتب עֶזְרָא הַסּוֹפֵר (باللاتينية: Esdras)

480-440 ق.م ، أحد أنبياء اليهود (وفقاً للعهد القديم وليس هناك تأكيد او نفي لذلك في القرآن والسنة ). كان موظفًا في بلاط إمبراطور الفرس (ارتحتشستا) ومستشارًا له في شؤون الطائفة اليهودية، وكان مُلِمًا بالتوراة ومُدَرسًا لتعاليمها، وكذلك كان كاتباً ماهراً للنصوص الدينية .

وقد تمكن عزرا من أن ينال عفو الإمبراطور عن اليهود وسماحه لهم بالعودة إلى القدس وإقامة حكم ذاتي لهم. فقاد مجموعة يهود المنفى في بابل إلى القدس وهناك فرض احترام التوراة وأعاد تعاليمها وطَهَّر المجتمع اليهودي من الزواج المختلط. ولهذه الأسباب يحتل عزرا الكاتب، مكانة عالية جداً في الإرث الديني اليهود وقصته مذكوره في ( سفر عزرا ) في العهد القديم ونجد في ملاحق الشروحات اليهوديه للمشناه والمعروفه باسم ( توسفتا ) תוספתא، نجد رأيًا يزعُمُ ان عزرا الكاتب كان يستحق أن تتنزل عليه التوراة لولا ان موسى، عليه السلام ،سبقه !

כי ראויה הייתה התורה להינתן על ידיו, אלא שבא משה רבנו וקדמו

(كان من الجدير أن تُعْطى التوراة من يديه ، لولا أنَّ موسى سَبَقَهُ )

كذلك هناك فصول محذوفة لسفر عزرا، يونانية الأصل، لم تُجمع مع العهد القديم وغير معترف بها من قِبل التيار الديني اليهودي والمسيحي الرئيسيين، وهي غير مُتَرجمة الى العبرية، وأحد هذه الفصول يحمل اسم " اسدراس " esdras ( عزرا باللاتينيه )وقد أدخل كاتبها عبارات تبشر بقدوم ابن الله .

اسدراس 2:46,2:47

‏46 :Then said I to the angel, What young man is he that sets crowns upon them, and gives them palms in their hands?

(عزرا يتكلم ) عندئذ سألتُ الملاك ، من هو ذلك الشاب الذي يضع التيجان على رؤوسهم ويضع البلح في أيديهم ؟

‏47 :So he answered and said to me, It is the Son of God, whom they have confessed in the world.

أجابني (الملاك) بأن (الشاب) هو إبن الله ، والذي إعترفَتْ به امام العالم ( حشود من الناس تجمهرت فوق قمة جبل صهيون ).



الاقتباس مأخوذ من صفحة 7 ، من هذا الرابط


على الرغم من أنه ليس هناك نص عند اليهود ينسب إلى العزير بنوة لله، إلا أن المعطيات المذكورة أعلاه، تُشَّكِل قرائن قوية على حدوث مُبالغة عند بعض الفئات اليهودية في السابق، بحيث أنها نَسَبت إلى العزير بُنُوة الى الله، ومن ثم اندثر ذلك الاعتقاد.

والآية الكريمة تذكر أن اليهود اعتادوا القول شفهيًا وقد يكونوا قد دونوا ذلك في كتبهم ولكن لم يصلنا منها شيئًا .

كذلك لَم يُذْكَر أن يهود المدينة في عهد الرسول (صلى الله عليه وسلم ) ابدوا اعتراضًا على الاية الكريمة أو تكذيبًا لها ، بل إن هناك رواية عن ابن عباس ذكرها الطبري في تفسيره، تفيد أن قسمًا من يهود المدينه كانوا يعتقدون بذلك. " أتى رسولَ الله صلى الله عليه وسلم سَلامُ بن مشكم, ونعمانُ بن أوفى, وشأسُ بن قيس, ومالك بن الصِّيف, فقالوا: كيف نتّبعك وقد تركت قِبْلتنا, وأنت لا تزعم أنّ عزيرًا ابن الله؟ فأنـزل في ذلك من قولهم: (وقالت اليهود عزير ابن الله وقالت النصارى المسيح ابن الله)، إلى: (أنى يؤفكون)."









الافتراء:

وعلى الرغم من كميّة الوثائق الكبيرة التي تثبت صحّة أسماء الأنبياء التوراتيّين ممن ورد ذكرهم بالتوراة و الأناجيل , سواء من قبل مخطوطات المؤرخين القدماء من من عاشر أولئك أم من قبل نقوشات الآثار و المعابد القديمة التي تنفي تعرّض اسماءهم للتحريف كما قام بتحريفها القرآن المحمّدي

و لا نعلم كيف استطاع ان يتكلم الانسان الاول - آدم - ان يتلو الشهادتين و يبني المساجد و يكون من المسلمين .. و الاديان السماوية لم يكن لها أي أثر قبل 7000 عام قبل ميلاد المسيح ومن المثبت علميّاً وتاريخيّاً أن الانسان الحالي وجد منذ 200,000 سنة ولم يستطع أن يكتشف كيفيّة استخراج المعادن وصناعة الاسلحة و الأدوات المعدنيّة و الأبنيّة واللغات المكتوبة والمقروءة الا في عصور متقدّمة نسبيّاً :

http://en.wikipedia.org/wiki/Homo_sapiens

http://humanorigins.si.edu/evidence/human-fossils/species/homo-sapiens

- و لاتهامهم بالتحريف ذلك له أهميّة خاصّة وذلك للهروب من مآزق جهله بدياناتهم و و ليتفادى أي تشكيك بالأخطاء التي يرتكبها أثناء النقل منهم , و بالرغم من ذلك الاتّهام فقد نقل منهم الكثير من قصصهم واساطبرهم حرّفيّاً



الرد:

أول خطأ في الفرية، هو عُمْر البشرية المزعوم، فالتقديرات تقول بأن عمر البشرية، بناءً على الهياكل العظمية التي عُثِرَ عليها، يعود الى 315000 سنة !

وللمزيد اقرأوا هذا الرابط


بالنسبة لنظرية التطور، فهي عبارة عن فرضية خاطئة علميًا لا دليل عليها، بل إن الحقائق العلمية تنقضها، والتطور هي كلمة تُقال وتُكتَب في المقالات العلمية فقط لتحاشي ذِكْر كلمة خالق لا غير. وحتى تتأكد عزيزي القارئ من تهافت خرافة التطور شاهد هذا الفيلم وستستغرب كيف أنَّ هناك علماء يحتضنون افتراضات التطور ويعجزون عن إثباتها حين يُسألون عنها، اليك الفيلم



ونحن لسنا بحاجة لإثباتات كي نصدق بوجود آدم عليه السلام أو اي نبي من انبياء الله ورسله ، نحن نصدق كل ما جاء في القرآن الكريم ، لأن القران كلام الله ومعجزته ومن يُشكك في ذلك فليتفضل وليثبت لنا ذلك عن طريق تأليف بكلماته شيئًا يضاهي آيات القرآن بلاغةً، واعجازًا علميًا واخبارًا عن الغيب ، فليتفضل !

مع ذلك اليك، عزيزي القارئ، بحثًا علميًا يؤكد ان البشر كلهم ينتسبون الى ام واحدة ( حواء ) ، هذه الدراسة قارنت الحمض النووي للمتوكوندريا (والذي يُورَّث فقط من الأم الى النسل)

لعشرات الأفراد من شتى الأعراق وخلُصت الى ان كل البشر ينتمون الى أُم واحدة ! اليكم رابط عن البحث








الافتراء:

يدّعي القرآن أن الله قد أرسل رسولا لكل أمة

" وَلِكُلِّ أُمَّةٍ رَّسُولٌ ۖ-;---;-- فَإِذَا جَاءَ رَسُولُهُمْ قُضِيَ بَيْنَهُم بِالْقِسْطِ وَهُمْ لَا يُظْلَمُونَ " سورة يونس : 47؛

" وَلَقَدْ بَعَثْنَا فِي كُلِّ أُمَّةٍ رَسُولًا أَنِ اعْبُدُوا اللَّهَ وَاجْتَنِبُوا الطَّاغُوتَ ۖ-;---;-- فَمِنْهُمْ مَنْ هَدَى اللَّهُ وَمِنْهُمْ مَنْ حَقَّتْ عَلَيْهِ الضَّلَالَةُ ۚ-;---;-- " سورة النحل : 36-

وَيَوْمَ نَبْعَثُ فِي كُلِّ أُمَّةٍ شَهِيدًا عَلَيْهِم مِّنْ أَنفُسِهِمْ ۖ-;---;-- وَجِئْنَا بِكَ شَهِيدًا عَلَىٰ-;---;-- هَٰ-;---;--ؤُلَاءِ ۚ-;---;--النحل 89 .

و هذا يناقض الحقائق التاريخية التي تخبرنا أن هذا لم يحدث . فلم يوجد اي مدّعي نبوّة أو رسل إلى أمم القارات الأفريقية والأوربية والأمريكية و الأستراليّة.ودول آسيا الوسطى و الشرقيّة فاقتصر أمر النبوّة على منطقة الشام وفارس ومصر..!!!

ومن المعلوم حسب القرآن أن محمد هو آخر الأنبياء وهنا نجد خطأ فادح أيضاً .

الأمّة في معاجم اللغة العربيّة :

الأُمَّة جماعة من الناس أكثرهم من أَصل واحد ، وتجمعهم صفات موروثة ، ومصالح وأَمانيّ واحدة ، أو يجمعهم أمر واحد من دين أو مكان أو زمان . يقال : الأُمة المصرية ، والأُمة السوريّة

http://www.almaany.com/home.php?language=arabic&word=أمة&cat_group=1&lang_name=عربي&type_word=2&dspl=0



الرَّد:

الادعاء اعلاه خاطئ لأنه بالإمكان العثور على اسماء انبياء في الحضارات البائدة، فعلى سبيل المثال، حين نقرأ تاريخ تأسيس الديانة المورمونية على يد المبتدع مُدّعي النبوة جوزيف سميث في أواسط القرن التاسع عشر .نجد أنه نقلها عن نُصّوص كُتبت قبل اكتشاف القارة الامريكية، وتذكر تلك النصوص اسماء انبياء بُعِثوا في تلك المناطق ، منهم النبي (مورمون) mormon و (مورموني) mormoni المبعوثين في فترة ما بين 322-421 بعد الميلاد .اليكم الإقتباس مع الترجمة

Book of Mormon was compiled from early records by two ancient American prophets, Mormon and Moroni, between AD 322 and 421,



تمَّ تجميع كتاب (ديانة) المورمون، من المخطوطات القديمة للنَبِيِّين الأمريكين القديمين (مورون) و (موروني) ما بين 322-421 بعد الميلاد



اليكم المصدر




من الناحية المبدأية، نحن نعتقد بأن الله أرسل انبياء الى كل قوم وقوم في أنحاء المعمورة لأن القرآن اخبرنا بذلك، والقرآن كلام الله ومعجزته الباقية، ولسنا بحاجة الى تأكيدات أثرية أو غيرها . إضافة الى ذلك، لم يكن معهودًا في التاريخ القديم توثيق الاشياء ، فاغلب الناس كانوا أُميين وطريقة التواصل ونقل المعلومات كانت بالأساس شفهيةً، وكانت الكتابة حكرًا على فئة المتعلمين القليلة.

كذلك، نحن نَعْلَم أن نُصُّوص الدين المسيحي واليهودي وهما العقيدتين الأقرب عهدًا الى الاسلام ، قد طالتهما يد التحريف والتغيير، فالأناجيل الأربعة متناقضة مع بعضها البعض، ومُحَرَّفة بحيث جعلت من المسيح عليه السلام إبنًا لله وفي بعض الاماكن الهًا. والعهد القديم جعل يعقوب يتصارع من الله ويهزمه! كذلك بحسب العهد القديم داوود وسليمان هما فقط مَلِكان وليسا انبياءً. هذه الفروقات موجودة في نصوص دُوّنت بأزمنة أقرب الى الاسلام من الحضارات القديمة. فاذا كانت اليهودية والمسيحية تحتوي على هذا الكم من التحريف فما بالك، عزيزي القارئ بأخبار الانبياء في الحضارات البائدة والأزمنة السحيقة، كم سينالها التحريف والتغيير! لذلك، من المحتمل جدًا أن تكون الآلهة في الديانات الوثنية عبارة عن انبياء بُعثِت في الزمن البائد ثم أضفى عليهم أتباعهم من بعدهم هالة من القداسة والأساطير التي قد رفعتهم فيما بعد إلى منزلة الإله. وقد يكون الحكماء في الحضارات القديمة عبارة عن انبياء خُلدت أقوالهم، مثل ؛بوذا وكونفوشيوس، وزرادشت.. دون ذِكْر نُبُوّاتهم.

بالنسبة للادعاء حول عدم كَون الرسول آخر مبعوث. فالرسول صلى الله عليه وسلم آخر الانبياء والمرسلين وبُعِث مع المعجزة الخالدة وهي القرآن الكريم. ولكن ذلك لا يعني عدم ظهور مُدَّعين للنُبُوِّة ودجَّالين، فحتى في حياة الرسول صلى الله عليه وسلم كان هناك من إدعى النبوة مثل مسيلمة الكذاب وغيره، وتكررت تلك الحالات بعد موته صلّى الله عليه وسلَّم، مثلما فعل جوزيف سميث المذكور أعلاه، وبهاء الله عند القاديانية وغيرهم من الأفّاكين.



الافتراء:

من بديهيات العلم و الطب عدم قدرة الكائنات الحيّة وخلاياها على العيش من دون طعام او شراب وان الاسلحة الحديدية و المعادن قد اكتشفت من بعد وجود الانسان الاول بسنين مؤلّفة ! وهذا ما يناقض بشدة قصّة ياجوج وماجوج الخرافية التي لا ترقى لأكثر من أن تكون اسطورة خياليّة لا يمكن أن يصدّقها عاقل بهذا العصر

يدّعي -الرسول - محمد انهم كانوا موجودون منذ عصر الانسان الأوّل " آدم " و ذلك بالحديث الصحيح الذي رواه البخاري (6529) وغيره الكثير ..:

عَنْ أَبِي هُرَيْرَةَ أَنَّ النَّبِيَّ قَالَ : أَوَّلُ مَنْ يُدْعَى يَوْمَ الْقِيَامَةِ آدَمُ ، فَتَرَاءَى ذُرِّيَّتُهُ ، فَيُقَالُ : هَذَا أَبُوكُمْ آدَمُ . فَيَقُولُ : لَبَّيْكَ وَسَعْدَيْكَ ، فَيَقُولُ : أَخْرِجْ بَعْثَ جَهَنَّمَ مِنْ ذُرِّيَّتِكَ . فَيَقُولُ : يَا رَبِّ ، كَمْ أُخْرِجُ ؟ فَيَقُولُ : أَخْرِجْ مِنْ كُلِّ مِائَةٍ تِسْعَةً وَتِسْعِينَ .

فهم لا يعيشون لا تحت الماء كما يقول بعض الشيوخ و لا حتى في جوف الأرض كما يقول بعضهم الآخر و ذلك لأنّهم كانوا يروا الشمس بكل يوم حسب الحديث الصحيح !!

عن أبى هريرة قال إنّ يأجوج ومأجوج يحفرونَ السّدَّ في كل يوم حتى إذا كادوا يَرَوْنَ شُعَاعَ الشمس، قال الّذي عليهم: ارجعوا فسنَحْفِرُهُ غداً .

الرَّد :

يتحدث الحديث الشريف الأول عن رؤية سيدنا آدم يوم القيامة لذريته وهم جميع البشر. أما كيف استنتج المستلحد ان الحديث يقول بأن يأجوج ومأجوج كانوا زمن آدم عليه السلام، فذلك من غرائب الأمور !



الحديث الثاني، لا يقول أن يأجوج ومأجوج يَرَوْن الشمس كل يوم ، كما يراوغ المستلحد ، بل يقول بانهم يحفرون في السَدّ كل يوم حتى يقتربوا من خرقه ورؤية شعاع الشمس ، لكنهم يتوقفوا عن الحفر في اللحظة الأخيرة.

لا القرآن الكريم ولا السنة النبوية أخبرا عن مكان يأجوج ومأجوج، فالموضوع من عِلم الغيب ولا أحد يعرف مكانهم.

الافتراء:

قال محمد او الله القرآني ذاته

سُبْحَانَ الَّذِي أَسْرَىٰ-;---;-- بِعَبْدِهِ لَيْلًا مِنَ الْمَسْجِدِ الْحَرَامِ إِلَى الْمَسْجِدِ الْأَقْصَى الَّذِي بَارَكْنَا حَوْلَهُ لِنُرِيَهُ مِنْ آيَاتِنَا ۚ-;---;-- إِنَّهُ هُوَ السَّمِيعُ الْبَصِيرُ ﴿-;---;--1﴾-;---;--

كيف بني المسجد الأقصى بالقدس ؟؟! بزمن محمذ !! الا يلزم لبناء ذلك المسجد وجود مسلمين بتلك المنطقة حينها ؟؟ !!

المسجد الأقصى لم يكن موجودا في وقت محمد.

فتاريخيّاً لا وجود لذلك المسجد بل كان معبد يهودي يدعى هيكل سلبمان قام الرومان بتدميره عدة مرّات وكانت اخر مرّة دمّروه بها بالعام 70 م حتى أعاد بناءه الخليفة الأموي عبد الملك بن مروان كمسجد عام 691 م، أي بعد موت محمد بحوالي ستين عاما !!

http://ar.wikipedia.org/wiki/هيكل_سليمان

وقد أخرج البخاري في صحيحه بالسند إلى أبي ذر قال: قلت: يا رسول الله، أي مسجد وضع في الأرض أول؟ قال: " المسجد الحرام " قال: قلت: ثمّ أي؟ قال: " المسجد الأقصى " قال: كم كان بينهما؟ قال: " أربعون سنة ،

قول الشيخ ابن عاشور في كتابه التحرير والتنوير: فهذا الخبر قد بين أن المسجد الأقصى من بناء إبراهيم لأنه حدد بمدة هي من مدة حياة إبراهيم - عليه السلام- وقد قرن ذكره بذكر الحرام مستدلّاً على ذلك من الآية 127 بسورة البقره : " و إذ يرفع إبراهيم القواعد من البيت وإسماعيل ربنا تقبل منا انك أنت السميع العليم . وبعض الأحاديث الأخرى " .

- أورد ابن حجر في الفتح ، كتاب أحاديث الأنبياء : " إن أول من أسس المسجد الأقصى آدم عليه السلام، وقيل الملائكة، وقيل سام بن نوح عليه السلام، وقيل يعقوب عليه السلام، وقال كذلك: وقد وجدت ما يشهد ويؤيد قول من قال: إن آدم عليه السلام هو الذي أسس كلا المسجدين، فذكر ابن هشام في " كتاب التيجان " أن آدم لما بنى الكعبة أمره الله بالسير إلى بيت المقدس، وأن يبنيه، فبناه ونسك فيه.

وهذا التناقض و التضاد أثار استهجان الكثير من الشيوخ و منهم الدكتور عدنان ابراهبم مما جعله يصرّح بجرأة عن تلك التناقضات رغم ورودها في الصحيحين :

انظر :

http://youtu.be/4TGUYulnbOU

الرد :

من يقرأ الافتراء، يجد ان المفتري وضع الكذبة وبنفس الوقت دحضها ، بإبرازه للاحاديث الشريفة التي تُبَّين أن المسجد الأقصى كان موجودًا قبل زمن الرسول صلى الله عليه وسلم بدهورٍ كثيرة ولا حاجة للإسهاب في الرد على هذه الافتراء المتهافت .



الافتراء:

القرآن أخذ الكثير عن معتقدات وطقوس المشركين و الأولين وخاصة اليهود منهم وضعها في قرآنه

كاسم الله الاله القمري عند المشركين - سين - وبيته الحرام - الكعبة – وتعابير ”الله أكبر". "اقرأ وربك الأكرم” وتعبير “ أحسن الخالقين "... والذي كان سبب ارتداد عبد لله بن ابي السرح عنه وكشفه احتياله و دجله –راجع قصة ارتداده - وهي تعابير تفاضل بين الله –القمر- وبقية الآلهة التي لا ترقى إلى مستوى الإله الأكبر

راجع : http://www.alzakera.eu/music/vetensk...toria-0141.htm

وشعاره الهلال الذي كان يرفرف على أعلام قريش و فوق معابدهم وامر محمد بوضعه نفسه على معابد المسلمين واحياناً بجانبه احدى بناته النجوم –عشتار- !! و تقويمهم القمري و تقديس ماء زمزم وتوجيه الصلوات و الدعاء نحو الكعبة وشفاعتها ، والحج السنوي و الطواف. وتقبيل الحجر الأسود وتقديسه والتضحية إلى الله وقذف الجمرات والصيام شهر رمضان وتحريم الحرب في الاشهر الحرم وتقديم الصدقات إلى الفقراء . حيث كانوا المشركون بعبادة الآلهة الشمس وبناتها " النجوم" مع الله " القمر " الاله الوثني الذي يعتبره المسلمون من صنع البشر اخذه محمد ليجعل منه الها حقيقيّاً للمسلمين بعد دمجه بآلهات اخرى من معتقدات الأولين وخاصةً اليهود كالحجاب – النقاب – تحريم تناول لحم الخنزير – طريقة الصلاة - قبعة الصلاة – الطهور ... الخ ونجد ذلك جليّاً بالحديث الصحيح الوارد عن عائشة الذي يوضح بشكل واضح دجله و اخذه من تعاليمهم , -عن عائشة قالت" يا رسول الله هذه يهودية لا نصنع إليها شيئا من المعروف ,إلا قالت: وقاك الله عذاب القبر .قال : " كذبت يهود ، وهم على الله كذب ، لا عذاب دون يوم القيامة " .قالت : ثم مكث بعد ذلك ما شاء الله أن يمكث ، فخرج ذات يوم بنصف النهار مشتملا بثوبه محمرة عيناه ، وهو ينادي بأعلى صوته : "أيها الناس استعيذوا بالله من عذاب القبر ، فإن عذاب القبر حق " .رواه أحمد ، ورجاله رجال الصحيح

http://library.islamweb.net/newlibra...k_no=87&ID=799

التي ثبت تاريخياً أنّها كانت موجودة قبله بآلاف السنين فأخذ منهم كل شيء مع انّه اتّهمهم بالتحريف !!!



الرَّد:

بدأت الرسالات النبوية من آدم عليه السلام ، ثم استمرت بعثات الانبياء والرسل حتى ختمتها بعثة الرسول صلى الله عليه وسلم. كل الانبياء كانت رسالتهم واحدة، عبادة الله الواحد الاحد خالق الكون والدعوة الى فعل الخير ونبذ الشَّر . على مدار التاريخ حَرّف الناس مفهوم الالوهية وشوهوا طقوس العبادة، فاشركوا عبادة الأصنام مع عبادة الله وصارت عبادتهم مشوّهه، والاسلام أتى بالصيغة الأصلية لكل الاديان ، صيغة التوحيد. والحَجّ في جَزيرة العرب عِبادة قديمة ترجع إلى عهد بناء البيت الذي جعله الله أول بيت وضع للناس، والذي رفع قواعدَه أبو الأنبياء إبراهيم ـ عليه السلام، وأمره أن يُسْكِنَ أسرته الصغيرة عنده، وأن يؤذِّن في الناس بالحجِّ استجابة لدعاء ربِّه أن يجعل أفئدة من الناس تهوي إليه ويرزُق أهله من الثمرات، وبهذا أصبح الحجّ شريعة متّبعة وموسِمًا حرَص العرب عليه ليشهدوا منافع لهم ولكنهم خالفوا بعض مناسك الحج وابتدعوا فيها، وذلك حين ظهرت الوثنية وعبادة الأصنام في الجزيرة العربية. فكانت بعثة الرسول محمد صلى الله عليه وسلم لتصحح التحريف الحاصل وترجع العبادة الى أصولها. بناء على ذلك ، فادعاء ان الاسلام نقل عن المشركين عبادتهم هو محض تخريف وافتراء لأن عادات الجاهلية كانت مُحَرَّفة عن عقيدة التوحيد التي كانت دائمًا رسالة الله الى الأقوام.



بالنسبة للهلال :

 فاتخاذ الهلال شعاراً للمسلمين لا أصل له في الشرع ، ولم يكن معروفاً في عهد النبي صلى الله عليه وسلم ولا عهد خلفائه الراشدين بل ولا في عهد بني أمية ، وإنما حدث بعد ذلك. ويقال أن اتخاذ الهلال سببه أن المسلمين حين فتحوا بعض بلاد الغرب وفي كنائسهم يعلو فيها الصليب وضع المسلمون الهلال لتمييز اماكنهم عن الكنائس فانتشر لذلك ، وعلى أية حال ، لا علاقة بين اتخاذ الهلال شعارًا وبين وجوده في معتقدات شركية سابقه ، فالاسلام دين توحيد يرفض الشرك والكفر وعبادة المعبودات.

-

- الافتراء:

. ألإسكندر الأكبر (356-323 ق.م).

كان الإسكندر الأكبر ملك ماكدونية (336-323 ق.م) وكان قائدا عسكريّاً قويّاً بارعا في الحروب. أسس إمبراطورية يونانية عظيمة في فترة قصيرة من الزمن قبل موته في ريعان شبابه بالغا من العمر 32 عاما. يدعوه القرآن ذي القرنين، وهو الاسم الذي أطلقه عليه المسيحيون واليهود في ذلك الحين لأنه كان ملك الشرق والغرب ] سفر عزرا الإصحاح 1 _ سفر دانيال : الإصحاح 6 _ و سفر أشعيا، : الإصحاح 44 - 45 [

أكتشفت عملات نقدية قديمة بها صورته بقرنين على رأسه وهذا هو السبب المرجح لاطلاق هذا اللقب عليه . يدّعي القرآن أنه كان عبداً بارا لله، وأن الله كان يحادثه مباشرة (الكهف 18: 83-98). وحتّى هذا ما قالته اسفار العهد الفديم في حين ان التاريخ يقول بعكس ذلك ، حيث أن الإسكندر الأكبر لم يكن رجلا تقيا. في الواقع، هو لم يؤمن بالله ولم يعبده، إذ كان يعبد آلهة وثنية مختلفة، وكان داعرا. وقد ارتكب أشد الكبائر الإلهيّة إذ ادعى أنه إله وطلب من الناس أن يعبدوه.و تقول الاثباتات التاريخية المكتشفة أنّه إدعى في مصر أنه ابن الإله آمون الذي عبده المصريون في ذلك الزمان، وعبده المصريون كإله. كان من المعتاد أن يُمثل الإله الوثني آمون بخروف ذي قرنين.

وبغض النظر انه لم يثبت تاريخيّاً أن قام أحد باكتشاف قارة امريكا " اقصى مغرب للأرض " كما يدّعي بعض الشيوخ و مفسري القرآن في محاولة بائسة لتفسير الخطأ العلمي الفدح في معضلة تفسير غروب الشمس بالعين الحمئة وبغض النظر بان القول بأن للشمس مغرب و مشرق على الأرض امر ينفي بشدة دورانها " و ذلك قبل عام 1498 م من قبل مكتشفها الأوّل كريستوفر كولومبوس ..

الرد :

الواضح من آيات القرآن الكريم ان ذَا القرنين كان قائدا عظيمًا مدججًا بالجند والعتاد ،مؤمنًا بالله واليوم الآخر ، وكان مُلهمًا من الله، وقد يكون نبيًا، وعلى أية حال ، فالآيات تقرر بأنَّ ذَا القرنين عمل على احقاق الحق وابطال الباطل. ولم تذكر آيات القران الكريم ولا السنة النبوية هوية ذي القرنين.

كل التكهنات والفرضيات حول هوية ذي القرنين تبقى كلها دون سند او دليل. فمن يفترض ان ذَا القرنين هو إسكندر المقدوني فهو يفترض افتراضًا من عنده ولا علاقة لذلك بالقرآن الكريم او بالسنة الشريفة. لاحظ عزيزي القارئ المحاولة البائسة لأعداء الاسلام للطعن في آيات القرآن عن طريق الاستشهاد بأشياء لا علاقة لها، لا بالقران الكريم ولا بالسنة الشريفة!

أما بما يتعلق بشبهة العين الحمئة فالافتراء مضحك وسخيف، وذلك لان الآية الكريمة تقول :" حَتَّىٰ إِذَا بَلَغَ مَغْرِبَ الشَّمْسِ وَجَدَهَا تَغْرُبُ فِي عَيْنٍ حَمِئَةٍ وَوَجَدَ عِندَهَا قَوْمًا ۗ قُلْنَا يَا ذَا الْقَرْنَيْنِ إِمَّا أَن تُعَذِّبَ وَإِمَّا أَن تَتَّخِذَ فِيهِمْ حُسْنًا" (الكهف 86)

فالآية واضحة وتقول ان ذَا القرنين وصل مكان في عين ماء طينية كبيرة وتزامن ذلك مع وقت غروب الشمس فرأى الشمس تغرب داخل مجمع الماء الكبير ، تمامًا مثلما نرى الشمس تغطس داخل البحر وقت الغروب!

و المقصود بمطلع الشمس هو نفس المعنى بمغرب الشمس ، هو المكان الذي وصل اليه ذو القرنين وتزامن ذلك مع شروق الشمس او غروبها ، فالمعنى واضح وبسيط، وكيد الكافرين وحقدهم في ضلالٍ بعيد



المقالات الأكثر رواجا

معاني كلمات القرآن الكريم - سورة مريم

بسم الله الرحمن الرحيم نرفق لكم أخوتنا الكرام ملف كامل عن معاني الكلمات مع لطائف شرح و تفسير الآيات لسورة مريم في  القرآن الكريم يمكن تحميلها عبر هذا الرابط

الرد المبين على الإفتراء في مقالة أخطاء القرآن العلمية

تجدون في أسفل المقالة رابط تحميل ملف نصي لردود على مجموعة أكاذيب وافتراءات ضخمة جمعها اعداء الاسلام، في سبيل الافتراء على القرآن الكريم والسنة النبوية الشريفة والادعاء انهما يحتويان على أخطاء علمية. اسم مجموعة الافتراءات والأكاذيب " أخطاء القرآن العلمية والردود الصلعمية الفاشلة عليها " وقد أبقيت على كل افتراء واتبعته بردٍ يليه . راجيًا أن يكون ذلك في ميزان حسناتي ، ولا تنسوني من دعائكم (محمد سليم مصاروه - صيدلي وماجيستير في علوم الأدوية ) للتحميل انقر هنا

الرد على الأكذوبة القائلة بأن علماء المسلمين كانوا ملحدين

الرد على الأكذوبة القائلة بأن علماء المسلمين كانوا ملحدين عن صفحة " الباحثون المسلمون " يروج الملاحدة و أعداء الحضارة الإسلامية الإفتراءات ويزعمون أن أشهر العلماء الذين نفتخر بهم كانوا ملحدين ، وطبعاً ذلك غير صحيح وإليكم توضيحات حول أبرز الشخصيات في تاريخ الإسلام المنير الفارابي وُلد سنة 260 هجرية ولُقِّب ب” المعلم الثاني ” نسبة للمعلم الأول أرسطو .. وهو شارح مؤلفات أرسطو المنطقية … وصاحب كتاب “الآثار العلوية” والمنافح عن عقيدة التوحيد .. والذي قضى عمره زاهدا متقشفا ليتفرغ لتأصيل فلسفة التوحيد، وواجب الوجود وحين مات صلَّى عليه سيف الدولة ابن حمدان .. يقول الفارابي ” معرفة الحقائق القصوى كلها مصدرها الله والفيلسوف يتلقى الحقائق بواسطة العقل الفعال فتكون طبيعتها عقليه وليس حسية، أما الرسول فتأتيه المعارف مُنَّزلة من عند الله بتوسط الملك جبريل " ويضع الفارابي شرطا جوهريا في مدينته الفاضلة وهو الإيمان بالله الواحد الأحد لكل أبناء المدينة ..  فكيف يُقال عن هذا أنه ملحد ؟..!! ابن سينا وُلد سنة 370 هجرية، وكان والده شيعياً إسماعيلياً .. ولُقِّب ب ” الشي

الإعجاز العلمي للقرآن الكريم في ذِكْر العلاقة بين المني المقذوف وبين فقرات الظهر وأضلاع الصدر

الإعجاز العلمي للقرآن الكريم في "ذِكْر العلاقة بين المني المقذوف وبين فقرات الظهر وأضلاع الصدر "  {فَلْيَنظُرِ الْإِنسَانُ مِمَّ خُلِقَ (5) خُلِقَ مِن مَّاء دَافِقٍ (6) يَخْرُجُ مِن بَيْنِ الصُّلْبِ وَالتَّرَائِبِ (7)} سورة الطارق    بقلم : محمد سليم مصاروه   صيدلي وماجيستير في  علوم الأدوية    تذكر آيات القرآن الكريم وجود علاقه بين المني المقذوف ( ماءٍ دافقٍ) وبين عظم الظهر ( الصُلْب ) وأضلاع الصدر  ( الترائب ) ، مضى على نزول آيات الذكر الحكيم 1440 عاماً وفقط قبيل سنوات اكتشف العلم الحديث أن العظم يؤثر على انتاج المني في الخصيتين وأن فقرات الظهر تحتوي على أعصاب تتحكم  بقذف المني .   كان يعتقد سابقاً ان وظيفة العظام تنحصر في دعم الجسم وتثبيته إلى أن كُشِفَ عن حقيقة افراز العظم للهورمونات ولذا فقد أضيف مفهوم جديد للهيكل العظمي وهو أنه غده كبيرة تؤثر على أماكن موجودة خارج العظم تفرز خلايا العظم تفرز هورمون الاوستوكالسين osteocalcin والذي يؤثر على خلايا " لايدغ" Leydig cells الموجودة في الخصيتين وهي بدورها مسؤولة عن انتاج الهورمون الذكري التوسترو

الرد على الإفتراء القائل بأن هناك أخطاء حسابية في توزيع الميراث في القرآن -العول

الرد على الافتراء القائل بأن هناك أخطاء حسابية في توزيع الميراث في القرآن -العول بقلم محمد سليم مصاروه يفتري المستلحدون واعداء الإسلام على القرآن الكريم ويزعمون أن القرآن فيه أخطاء حسابية حول تقسيم حصص الميراث ويتمادون في كذبهم فيَدَّعون أن القرآن من تأليف محمد (صلى الله عليه وسلم) وأنه أخطأ حسابياً في تحديد الحصص وذلك لأنه في حالات مُعَيَّنة يكون مجموع حصص الورثة أكثر من ١٠٠٪؜ وفِي حالات أخرى يكون أقل من ١٠٠٪. والحقيقة أن من يشكك في القرآن الكريم فهو أكثر من مدعو إلى أن يحاول أن يكتب شيئًا مثل القرآن الكريم وليقدم لنا  إبداعاته! على كل حال، حدَّدت آيات القرآن الكريم مقدار حصص الوارثين المحتمل وجودهم على الغالب أثناء تقسيم الميراث، فمثلاً ترث الأخت نصف مقدار الأخ الشقيق ولكن هناك الكثير من الاحتمالات لوجود عدة أنواع من الورثة في نفس الوقت مثل (أخ، أخت، عّم، جد حفيد وكذا) وبطبيعة الحال ليس من المعقول افتراض تفصيل آيات القرآن الكريم لكل الحالات التي فيها تراكيب مختلفة من الوارثين، وإلِّا لصار القرآن مُجَلَّدات من الحسابات والمعادلات الرياضية وعندها سيكون سُمْكُه مث

من هو العزير الذي قالت عنه اليهود أنه ابن الله ؟

مقالة بقلم : محمد سليم مصاروه من هو العزير الذي قالت عنه اليهود أنه ابن الله ؟ هو عزرا بن يوسف او عزرا الكاتب עֶזְרָא הַסּוֹפֵר  (باللاتينية: Esdras) 480-440  ق.م هو أحد أنبياء اليهود (وفقاً للعهد القديم وليس هناك تأكيد او نفي لذلك في القرآن والسنة ) كان موظفًا في بلاط إمبراطور الفرس (ارتحتشستا) ومستشارًا له في شؤون الطائفة اليهودية وكان ملماً بالتوراة ومدرساً لتعاليمها وكذلك كان كاتباً ماهراً للنصوص الدينية وقد تمكن عزرا  من أن ينال عفو الإمبراطور عن اليهود وسماحه لهم بالعودة إلى القدس وإقامة حكم ذاتي لهم، فقاد مجموعة يهود المنفى في بابل إلى  القدس وهناك  فرض احترام التوراة وأعاد تعاليمها وطهر المجتمع  اليهودي من الزواج المختلط، ولهذه الأسباب يحتل عزرا الكاتب مكانه عاليه جداً في الإرث الديني اليهود وقصته  مذكوره في ( سفر عزرا ) في العهد القديم ونجد في ملاحق الشروحات اليهوديه للمشناه والمعروفه باسم ( توسفتا ) תוספתא نجد رأياً يُزعم ان عزرا الكاتب كان مستحقاً لان تتنزل عليه التوراه لولا ان موسى عليه السلام سبقه ! כי ראויה הייתה התורה להינתן על ידיו, אלא שבא משה רבנו וקדמו ك